Torts Neg

You might also like

Download as docx, pdf, or txt
Download as docx, pdf, or txt
You are on page 1of 82

Torts is a double-tested subject.

And on the essay exam, on the MEE for the torch questions,
you are going to want to lay out for the grader
the elements of whichever tort, you are analyzing.
You're going to write down the elements of battery,
or the elements of negligence, or the elements
of a strict liability claim for a defective product.
Those elements tend to be phrases or full sentences.
There's no one word in those phrases or sentences that's
more important than any other word.
All the words are important.
So you've got to learn the whole phrase and the best way
to learn the whole phrase is to write down the whole phrase.
And so what I have done-- and BARBRI
is very accommodating in allowing
me to pursue my own best instincts
as to how to teach the material-- what I have done
is basically decided that you should just take
an old-fashioned set of notes.

There is another block of testable material on torts


for the UBE, and that material relates
to privacy and defamation.
The tort of defamation and the four different privacy causes
of action are dealt with in a module that
is available in a different recording
and that is something that you can access online.

Observation number one is that in the world


of intentional torts, the hypersensitivity of a plaintiff
should be ignored.
Plaintiff's hypersensitivity should be ignored.
Now, at the risk of pissing you off,
remember what I said about note-taking?
So you hear me say those words, the easiest thing in the world
to do is just write down those words.
Probably you've already done it.
Hypersensitivity is ignored.
But if you were to pause and say, what does he mean ignored?
Well, disregarded it, hypersensitivity.
So extreme sensitivity is disregarded.
The few seconds that it took you to rephrase it in your brain
actually imprints it on your brain.
So yeah, extreme sensitivity is disregarded.
Eccentricity, not taken into account.
All kinds of different ways you can make the principle.
So the bottom line is whenever you have an intentional tort
problem, you go ahead and you assume that the plaintiff is
a reasonable person.
Only if a reasonable person would
be able to establish that claim should
you allow a tort recovery.
Let me move on to my second overview observation.
In the world of intentional torts,
there are no incapacity defenses.
There are no incapacity defenses.
Ignore lack of capacity.
Again, you can reword it.
You should reword it.
Rewording is good.
What I mean by this may require a teeny bit of expansion.
I think everybody is aware by the time
they get to the point of a bar exam
that there are many incapacities in American law.
I will use as my first example or illustration
the incapacity of being a child.
10-year-old, you're a 10-year-old kid.
You lack the capacity, the legal capacity to do almost anything.
As a 10-year-old, you can't get lawfully married.
As a 10-year-old, you cannot drive a car.
They will not give you a license.
As a 10-year-old, you cannot execute a valid commercial
contract.
If you try, your commercial contract can be disavowed.
Minors lack capacity.
But if a 10-year-old kicks you in the leg,
you can sue the little bastard and you
will succeed because incapacity is not
taken into account in the world of intentional torts.
If a 10-year-old locks you in the closet,
again, good claim for false imprisonment.
This rule applies to every type of incapacity
they could put on the exam.
That means insane people, liable for intentional torts;
developmentally disabled people, liable for intentional torts;
intoxicated people, liable for intentional torts.
Put it the other way around.
You read an intentional tort question.
One of the choices says, defendant
is not liable because he lacked legal capacity.
Eliminate that choice right away.
That choice just drops out.
It can't be correct.
Third and final introductory comment,
the doctrine of transferred intent.

Every intentional tort has as a required element


that the defendant be acting intentionally.
In law school, you spend a lot of time exploring what
the law, what the legal concept of intent actually means.
But on the bar exam, they don't really go very deeply in it.
If someone is acting intentionally,
if they're acting on purpose, if they're acting deliberately,
a more formal definition of intent
is to say that you're acting with the desire
to produce the legally forbidden consequence.
If the tort is false imprisonment,
the forbidden consequence is confinement.
So you desire to cause confinement
if the tort is intentional infliction
of emotional distress, the forbidden consequence is
that the person is emotionally bummed out
so you desire to bum them out.
You're acting with the purpose or the desire
to produce the legally forbidden consequence.
However, there is a curlicue, a footnote, a parentheses,
and that's transferred intent.
If at the beginning of a scenario,
a defendant starts out with the desire
to commit one forbidden consequence, one type of tort
against a particular victim, and if a different consequence
results, a legally forbidden consequence
of a different variety comes to pass,
or if a different victim is affected,
we're still going to hold that defendant
liable for an intentional tort.
I see my enemy John.
Wanting to hurt him, I pick up a rock and I throw it at him
but my aim is off.
I missed John and I hit Patricia, a passer-by,
a pedestrian who's walking behind him.
I will be liable for battery to Patricia.
I want to hit John in the head with a rock,
I throw the rock at John.
I missed John but the rock goes whizzing by Patricia's face.
She doesn't get hit but she's very scared.
She has a brief moment where she thinks I'm going to get hit.
I would be liable to Patricia for the tort of assault.
Now, the thing to understand about transferred intent
is that the title of this doctrine
is just a legal fiction.
You cannot reason from the label that we put on most things
in the law.
Transferred intent is not about intent.
If you're trying to hit your enemy John with a rock
and you missed and you hit Patricia,
if you were to explain that to your grandma,
she would say, oh so you hit Patricia by accident.
She'd be exactly right.
But the law doesn't treat it as an accident.
It's a legal fiction.
But it's an important legal fiction
we're going to call out a battery,
and we will see transferred intent come up later on as we
work through the material.

So the very first of these torts that we're going to consider


is the tort of battery.
And for battery, I've got to two testable elements.
I'm not going to keep listing intent.
Intent is an element for all of them,
it would be a waste of my breath to list that
over and over again.
So two additional testable elements.
Number one, the defendant must cause a harmful or offensive
contact.
The defensive must commit or cause
a harmful or offensive contact.
Element number two, that contact has
to be with the plaintiff's person.
So let's look at each element somewhat more closely.
Harmful or offensive.
What can I ask about?
Can they do anything with the idea of harmful?
Can they try to mess with you?
Can they give you an issue where you have to think about,
is it harmful?
In my opinion, no, they cannot.
The concept of harmful is self-evident.
It means exactly what it sounds like, there's
nothing tricky there.
A contact is harmful if it causes you to bleed,
if it breaks your bones, if it sends you to the hospital,
if it leaves you paralyzed, if it leaves you blind,
if it leaves you dead, then it's harmful
and they can't ask about it.
I put this one in a category that I call too easy to test.
What are they going to do?
Are they going to write a question,
Dave walked up to Pete and shot Pete in the head.
Discuss.
Discuss.
Were you going to get that wrong?
Yeah, that's a battery.
Yeah, you know that.
If you're not able to get that question right,
not only should you not get a law license,
you shouldn't even get a driver's license.
So it's not going to--
don't worry about it.
Harmful is harm.
What about offensive?
Can they mess with you?
Can they give you something where
it's like, I don't know if it's offensive?
Well, yeah, they can, and here's the deal.
In order to determine if something is offensive,
just substitute the word unpermitted.
A contact is offensive if that contact is unpermitted.
But unpermitted by whom?
Well, now, we come back to that first overview,
that first introductory proposition.
Unpermitted by a person of ordinary sensitivity,
extreme or hypersensitivity is not taken into account.
Let me give you an example.
I love my examples.
I'm a very concrete thinker.
I'm not saying that for the purposes of this presentation.
I'm not saying that because BARBRI told me to say it.
It's true, I'm a very concrete thinker.
When I talk to my colleagues at GW,
and they talk to me in the abstract world,
I'm like can you make that?
Can you give me an example?
So here's an example.
I have a female college student, an undergraduate.
I'll give her a name, I'll call her Patricia.
Her name begins with P. Why does her name begins with P?
She is going to be the plaintiff.
Where is Patricia at the time of this story?
In the university's main library reading room,
big cavernous room.
She's got books and papers set up.
She's got a laptop open.
She's trying to write a paper for a course
that she is enrolled in.
As she's working, who enters that reading room but Dave.
Another student at the same school.
Noticed that his name begins with the letter D. Not relevant
to the story but I like to imagine that Dave
is on the football team.
I picture him as like a really big guy, like 6 foot 3,
like 250 pounds.
It's like hulking guy, rarely seen in the university library,
by the way.
He looks over.
He looks around and scans the room, he sees Patricia.
He makes his way straight to Patricia's location.
He walks through that whole library.
Sits down next to her.
And he says, Patricia, I've been looking for you all day
because you see, there's some I've been meaning to tell you.
She's like, Dave, not now.
I'm working.
He's like, no, this will wait.
You see, I really, really like you.
But as he says this, he reaches out and he strokes her hair.
I really, really like you.
She sues for battery, as well she should.
Is she going to win?
Well, that will depend on whether or not--
there was definitely a contact, question
is was it an offensive contact?
You got to ask yourself, was that contact unpermitted?
Would an ordinary person not permit that contact
or is that something that an ordinary person would
be OK with?
The answer is it's unpermitted.
Why?
Because it was a sexual touching.
It was harassment.
It was to put the most blunt word that I can think of on it,
it was a grope.
He groped her.
Fortunately, it was not a more intimate part of the body
but that is just as unacceptable as any other grope.
Keep your hands to yourself.
Don't be touching.
And I use that example because I think an unwanted sexually
motivated touching is kind of like the poster child
for an offensive touching.
I mean, what could be more offensive than that,
to be touched in a sexual way when you don't want
to be touched in a sexual way?
So that's a good example of that,
although I do want to say one further thing
about the hypothetical.
Even if I were to take out the sexual harassment
component of that hypothetical, I
think that would still be an offensive battery.
Let me modify the hypothetical.
Let's say it's the first time I taught a course at GW.
I'm in my big classroom.
I got my 100 students.
Everybody sit in the big tiered seats,
waiting for the first Socratic cold call.
But I'm like, I'm going to be friendly.
Walk around the room and say hello to people.
Hi, how are you doing?
Welcome to law school.
Hope you're psyched to be here.
But as I greet each person, I stroke them on the hair.
Hi, welcome to law--
what the hell is that?
That's not normal.
That's like battery city.
Don't do that.
We don't pet each other.
When you start petting strangers,
you're going to expose yourself to legal liability.
So that's element number one.
Element number two in the tort of battery
is that the contact has to be with the plaintiff's person.
The plaintiff's person includes anything--
excuse me-- anything that the plaintiff
is touching or holding at the time that the event occurs.
If you're walking down the street
and you're carrying a shopping bag or a purse
or a handbag of any kind, a suitcase or briefcase,
somebody grabs that item out of your hand.
Even though they never touch your skin, that's a battery.
Your body includes anything you're touching, holding,
or connected to.
Many years ago, they had a question
on the multistate exam.
The woman had gone about horseback riding.
I don't know where this was supposed to be,
but I don't know, you can go horseback
riding in a lot of places.
She lost her way.
She sees a pedestrian.
She liens down from the horse and she says, excuse me,
can you tell me the way back to the stables?
The pedestrian, instead of answering her question,
slaps the horse on the ass.
They never explain why.
They just said that he did it.
This is the way they write the bar exam.
I don't know what's up with that.
In that question, what they were looking for
was that the rider, the woman on horseback,
could recover for the tort of battery
because the horse is considered part of her person.
It was a contact with her person.
One of the wrong answers said she can't recover because there
was no bodily contact.
Incorrect.
S She's on the horse.
The horse is part of her body.
In the eyes of the law, she's like a figure
from Greek mythology, half-woman, half-horse.
So that's what we have with respect to that element.
I want to put you to work.
Let me give you a very, what I hope
is a very straightforward example
of some of these principles.
I call my questions here micro-essays.
I'm going to give you a fact pattern.
I want you to write down a one word,
yes or no answer when I get to the end of the fact pattern.
And then a one-sentence explanation.
One sentence, keep it short, eight or 10 words max,
that's all you need.
Just to test if you know what you're doing.
You might want to open up a separate file
and do this in a different word document.
Open up a tab, open up another document.
You might want to do it on a piece of scratch paper.
It's your call.
Here's my fact pattern.
Dave is at a professional conference.
They take a break after a panel discussion in the morning.
He goes out in the lobby, he needs to use the bathroom.
He has no idea where the bathroom is located.
He's looking around, he's looking around,
and he can't find it.
But he notices another guy, a man named Pete.
Pete is somebody who works for the conference organizers.
Pete is the guy who checked Dave in earlier that morning.
That's how Dave recognizes him.
That's the dude who checked me in.
He's almost certainly going to know where
the bathrooms are located.
So Dave approaches Pete.
But Pete has got his back turned to Dave
and is engaged in conversation with somebody else.
So Dave walks up to him and taps him on the shoulder.
He's like, tap, tap, tap, excuse me.
Can you tell me where the men's room is located?
Tap, tap, tap.
Now, a few weeks earlier, Pete had
been diagnosed with an unusual but legitimate
psychiatric condition that causes
him enormous anxiety when he is tapped on the shoulders.
And so as a result of this tap on the shoulder,
he has a panic attack.
And he turns around and he sues Dave for a battery.
What I want to know is, will Pete win that battery claim?
Yes or no, and a one-sentence explanation.
If you said no, props to you.
I think that's a good job.
Let me explain why.
That's clearly not a situation-- there clearly
was bodily contact there and the contact was
clearly intentional.
It's not like Dave bumped into him.
He touched his shoulder on purpose.
On purpose.
So definitely a contact, definitely done intentionally.
Question is whether or not it was harmful or offensive.
Certainly not harmful, no bleeding,
no going to the hospital.
Was it offensive?
Well, it's only offensive if will
be unpermitted by a person of ordinary sensitivity.
And it turns out that Pete, the plaintiff in this case,
not a person of ordinary sensitivity.
In fact, he's got an unusual sensitivity.
He's got this weird psychiatric condition,
this quite extraordinary psychiatric condition
that gives him panic attacks when
people touch his shoulders.
In effect, I guess there is a moral to this story
and I think you can take that moral this way.
In a strange way, offensive batteries,
they're not about you.
They're really not about you and your own personal sensitivity.
They're really about kind of a blended standard
of what is socially acceptable.
And so in this case, that would have been no recovery.
I want to conclude with a final thought about battery
before we move on.
In all of the examples that I have used so far,
the contact takes place instantaneously
and then we are analyzing whether or not
it was with the plaintiff's body,
we're analyzing whether or not it was offensive.
But it doesn't have to take place instantaneously.
It could be that you set up a contact
to take place after a delay.
Imagine that you and I work together in the same office,
and I really dislike you.
In our office, I want you to imagine that there's a pantry.
You can leave your lunch in there.
There's a fridge, and you do, you
would order to save some money, you bring a bag lunch
from home.
And I know it's your lunch because it's
got your name written on it in Sharpie.
But I go in the fridge at 10 o'clock in the morning
and I take your lunch out and I unwrap your peanut butter
and jelly sandwich and I put poison in the sandwich.
Then I carefully rewrap it, and I put it back
into the brown paper bag, and I put it back in the fridge.
Two hours later at noon, you eat that sandwich,
you become violently ill.
You go to the hospital.
You have sustained a harmful contact,
the contact is the poison with your mouth.
I caused that.
I'm the one who put the poison in the sandwich.
That's a battery.
It doesn't have to be a punch in the nose.
It doesn't have to be a spit in the face.
It doesn't have to happen one-two,
it can be after a delay.
So with all of that, I think we're
free to move on to tort number two.
The second tort we're going to take up is the tort of assault.
Assault has two testable elements.
Number one, defendant must place plaintiff
in reasonable apprehension, reasonable apprehension element
number two, that apprehension must
be of an immediate battery.
Reasonable apprehension of an immediate battery.
Now, the phrase reasonable apprehension, which
is element number one, requires some explanation so let's
delve into that.
The phrase reasonable apprehension
appears in hundreds of cases, and it comes out
of the restatement of torts.
But it is in my opinion an ill-chosen word.
Let me explain what I mean.
In everyday speech, the word apprehension
usually is a synonym for fear, nervousness, anxiety.
You might conceivably say, if you
choose to use the word at all, I don't want to go to that club,
I'm kind of apprehensive about being there
because I heard there was a shooting there last month.
But if you were to pull up a dictionary online and look up
the word apprehension, you would discover
that there is a second meaning to the word,
and that second meaning is knowledge.
The word apprehension actually does mean knowledge as well as
fear.
So in other words, it is linguistically correct to say,
I apprehend that the capital of the state of New York
is Albany, I apprehend that Albany
is the capital of New York.
That doesn't mean you're afraid of Albany,
although perhaps you are, but it's irrelevant
whether you are in a context that just means
that you are aware of the fact.
To apprehend something is to know it.
Now, that is linguistically correct, but who the hell
talks like that?
The answer is nobody but pompous idiots
and the people who wrote the restatement of torts--
a significantly overlapping bunch.
I want to tell you something about the restatement.
Forgive me for going on what you may think of as a tangent,
but I think we're going to see this over and over again.
There are multiple restatements of torts,
the current one is Restatement Third,
but we continue to look at Restatement
Second for the definition of a lot of the causes of action.
Restatement Second is where we get this phrase
reasonable apprehension.
Restatement Second was written, published somewhere
around the 1970s.
It was mostly written by old guys.
Most of them were about 70, some of whom were 60.
So you do the math, these were people
who were born around 1900-1910, they were growing up
in the 19-teens, 1920s, they went to law school
in the 1920s.
And you know what?
They were mostly kind of privileged,
because who do you think went to law school in the 1920s?
You ever take a look at some of those old-timey class pictures
from your old law school in the 1920s?
It's very diverse, bunch of white guys.
So these were probably reasonably affluent,
reasonably socially upper class white guys.
And you know what?
They talked funny.
They were kind of using like this Great Gatsby
mode of speech.
They would be like, oh, Sebastian,
it's so marvelous to see you.
You should come to dinner tonight.
I apprehend that mother will be making chicken.
They weren't afraid of the chicken,
they just knew that mother was making chicken,
and that's the way they talk to each other.
When it came time for them to write the restatement,
those are the words, they used.
It's a pain in our rear end because they're using
vocabulary from a century ago.
But we're stuck with it.
Why am I going into this big long tangent?
Because the bar examiners love to play with the fact
that the word is a poorly chosen word.
Here's what you need to know.
Because apprehension means knowledge,
a plaintiff in assault must have knowledge, an awareness
that he or she is about to be touched,
he or she is about to suffer a battery.
You have to be aware.
You have to know that a touch is imminent.
Colloquially, I would say you have to see it coming.
If you are unaware--
what's another word for unaware?
If you are oblivious, then you cannot ever recover
for assault. If I walk into your bedroom and you are sound
asleep and I pick up a stick as if to clobber you,
and then I changed my mind, no assault because I have not
placed you in a reasonable apprehension,
because it means that you have to have knowledge and you have
no knowledge because you're asleep and you can't see me.
You can't see me, no knowledge, therefore
no reasonable apprehension.
On the other hand, you don't have to be afraid.
Even though in colloquial speech the word apprehension
could mean fear, there is no requirement
that you need to be afraid.
Imagine a small guy goes up to a big guy.
Small guy, 5 foot 5, 130 pounds, threatening a big guy.
Big guy is like some massive hulking NFL player.
Little guy picks up a stick and says, I'm going to hit you.
The big guy sees this.
He hears him say, I'm going to hit you.
He sees him waving the stick but he's not afraid.
He knows that if he does hit him, he can defend himself.
And therefore, you might say, well,
I don't know if he's in reasonable apprehension.
Well, he is in reasonable apprehension
because apprehension means knowledge.
Now, just FYI.
In case this were to come up in your practice down the road,
the Third Restatement finally gets
rid of this troublesome phrase.
The Third Restatement uses the phrase anticipation.
You must anticipate a battery.
You must anticipate an immediate or imminent battery.
Wow, what an improvement, yay.
It took them like 50 years to do it but, yay--
anticipation instead of reasonable apprehension.
But the word apprehension is in all the cases
because it's been around for a long time.
And so you may still see it on the exam, which
is why I have spent all this time warning you
away from what is a really stupid exam track.
Now, there is another aspect to this first element
that is worthy of note.
It's what I call the unloaded gun problem.
Unloaded gun problem.
Sometimes they will give you a fact pattern where a defendant
is threatening to commit a battery but cannot actually do
it.
It is a bluff.
It is an idle threat.
It is an empty threat.
Now, you don't have to write down all
three of those phrases.
Remember, you're not trying to take
a transcript of what I say.
Just pick the one that you like--
empty threat, idle threat, can't do it.
And why do I call it the unloaded gun problem?
Because that would be the iconic fact pattern.
Dave walks up to Pete, reaches in his pocket, takes out a gun,
and says, hey Pete, I'm going to shoot you right now,
10 seconds.
But there are no bullets in the gun.
No bullets in the gun means can't really
effectuate any battery.
Pull that trigger, there's not going to be any bodily contact.
It's an empty threat.
It's an idle threat.
What about that?
Is that an assault?
Well, it depends on what plaintiff knows.
It's all about the plaintiff's knowledge.
If the plaintiff knows the gun is unloaded,
plaintiff knows that there can be no battery.
If you know that there can be no battery,
then it's not an assault. If on the other hand plaintiff
is aware that the gun is loaded, if he has concrete information,
come out the other way.
That would be a battery.
What if the plaintiff has no information one
way or the other?
What if Dave, the man with the gun,
is a criminal who just jumped out from some shrubbery?
Says, I'm going to shoot you, but it's an unloaded gun.
He's a weird criminal.
He uses unloaded gun.
Now, what I would do is I would direct your attention back
to the adjective that we put in at the beginning
of our statement of the elements.
You notice I didn't say apprehension,
I said reasonable apprehension, reasonable.
The word reasonable is there for a reason.
It is sufficient to make out an assault
if someone in the plaintiff's position
would reasonably expect that a battery would happen.
You don't have to know to a certainty.
Is it reasonable to think that a dude holding a gun
could effectuate a touching by pulling the trigger?
In other words, is it reasonable to assume
that the gun is loaded?
I'm going to go yes.
Most guns are loaded.
Most people holding guns, and pulling them out,
and threatening you are not walking around
with unloaded weapons.
So yeah, I think that's reasonable
and I think in most cases, the plaintiff could recover.
So let me shift your focus if I can to element number two.
Element number two is about timing
but it actually plays out on the exam
in a slightly different way.
Element number two says that this knowledge
that the plaintiff possesses, this reasonable apprehension
or this anticipation, must relate to an immediate battery.
Stress word there is immediate.
Might want to go back and bold that or put it
in a big font-- immediate battery.
But here are the two testable propositions.
Number one, words alone lack immediacy.
There has to be some overt conduct.
There has to be some conduct.
Words alone lack immediacy.
If I walk up to you with my hands in my pockets,
I'm completely motionless.
My videographer by the way tells me
I should try to be motionless so I'm going to try to,
I'll do that for this hypo.
So I'm standing, I'm just completely motionless.
So this makes a very good camera shot, I don't know why,
like I'm dead.
So there I am, and I'm like in five seconds
I'm going to punch you in the face as hard as I can.
Oh, it's really going to hurt.
In five seconds, I'm just going to hit you.
Oh, it's down to three seconds now.
Two seconds.
Is that an assault?
The answer is no.
It's not an assault. It's not an assault
because it's not sufficiently imminent.
You might say to me, so not sufficiently imminent.
How could it not be imminent?
You said five seconds, three seconds, two seconds.
Sounds pretty imminent to me.
Well, the thing is that the legal system takes the position
that talk is cheap.
That if you're just running your mouth, it's just idle talk.
It's just chatter.
It's a bluff.
It ain't real until you make a move.
It ain't real until you engage in menacing conduct.
What constitutes menacing conduct?
Could be almost anything.
On the exam, it's almost always obvious.
I would say 75% of the time, the defendant has a weapon--
a gun, a knife, a rock, a stick.
If not that, he's drawing his hand back as if to strike you.
He's clenching his fist and shaking it in your face.
You need to conduct to make it immediate.
That's proposition number one.
Proposition number two.
Proposition number two is that even when
I have menacing conduct, words can destroy or negate
the immediacy that is inherent in the gesture.
So let's say I'm drawing my hand back as if to strike you.
But I say, as I do so, if you weren't my best friend,
I'd slap you silly.
If you weren't my best friend, I'd slap you silly.
In those circumstances, you will not
Extreme and outrageous conduct.
Let me talk about that for a couple of seconds.
In the first three torts, the law
sets out a forbidden consequence.
And any way you accomplish that consequence
is considered to constitute a tort.
We don't really care about the methodology.
You cannot touch me in an offensive way,
whether that is groping me in an intimate way,
whether that is spitting in my face,
whether it is throwing mud on me,
whether it is doing anything else that would constitute
offensive touching in the eyes of an ordinary person,
it doesn't matter.
But this tort is different.
It's not enough that you desire to emotionally distressed me.
You must do it through a particular mechanism.
You have to use a particular means or a particular tactic
to achieve your evil goal.
So I look at somebody and I was like, oh, there's John.
I want to distress John.
I'm only going to be committing a tort
if the way that I choose to distress him
is to use outrageous conduct.
What does that mean?
Restatement of tort has a definition.
It tells us that conduct is outrageous
when it exceeds all bounds of decency tolerated
in a civilized society.
It exceeds all bounds of decency tolerated
in a civilized society.
And you know what?
If you have an essay question on this tort,
try to remember and put that phrase in the exam booklet.
Say, plaintiff has to show outrageous conduct,
outrageous conduct is defined as, and then
use that definition.
And wow, that definition is something, isn't it?
You can sure hear the voice of those restatement authors
there.
Does that sound like it's from the 1920s?
Now, Sebastian, did you hear what Chauncy did last week?
He exceeds all bounds of decency tolerated
in a civilized society.
Oh, but you should come to dinner.
I apprehend mother is making chicken.
Exceeds all bounds of decency tolerated
in a civilized society.
I have never found that phrasing to be particularly helpful.
And let me tell you why.
And there may be a lot of reasons why but I'm convinced
one of the key reasons.
I mention I teach at GW.
Now, I've been at GW my whole career.
But I'm not originally from the DC area.
I grew up in New York.
You may have guessed that.
And once a New Yorker, always a New Yorker.
So I look at this phrase exceeds all bounds of decency tolerated
in a civilized society.
I'm a New Yorker reaction, I'm like,
that's a null set because to a New Yorker, to a New Yorker,
there's nothing that exceeds all bounds of decency.
We tolerate anything.
Come on, you get on in New York City subway,
there's some guy sitting across the way from you,
he's holding a severed human head.
What do you do?
Don't make eye contact.
He throws the head at you.
What do you do now?
Throw the head right back at him.
Hey, buddy, you forgot your head.
You leave the subway, there's dead bodies
all over the street, you're like, this is disgusting.
They forgot to clean up today.
Your license plate says, forget about it.
Nothing exceeds the bounds of decency that you tolerate.
Now, on the other hand, some of you, the vast majority of you,
you are not from New York.
Some of you are from wonderfully charming places.
You're from places like Minnesota, Kansas, Nebraska.
You go to your birthday party.
Somebody smeared the icing on your birthday cake.
You're like, this exceeds all bounds
of decency tolerated in a civilized society.
All right, if that's your--
if this is a helpful statement to you, good for you.
I'm going to try to be more concrete.
Let me give you one negative rule
and then let me give you some positive or affirmative
information.
The negative rule is simply this.
Mere insults are not considered outrageous conduct.
Mere insults.
Now, the word mere is there for a reason.
It means insults by themselves.
Insults can be packaged with other things such as threats
or other items that I'll mention momentarily.
But by themselves, just insults, not going to be enough.
Even if they are used by someone who is intentionally
desiring to produce distress.
I see John.
Oh, I don't know, maybe John is a student in my torts class,
and I decided, you know what, I'm really
going to distress John.
So I walk up to John, I decide to insult John.
I decide to insult him based on his appearance.
I go, dude, dude, I don't know if anybody's ever
told you this, but you are the ugliest person I've ever seen.
You are positively hideous.
I mean, man, I have never seen somebody who looks like you.
What did you get when you were born?
Did the doctor hit you in the face with a frying pan?
That is just some kisser you've got on you.
Could do me a favor?
For the next part of the class, could you
turn around and face the back of the classroom
so I don't have to look at you?
He's probably going to be a little bit distressed.
But distressed or not, it's not outrageous conduct.
It's not outrageous conduct because mere insults are not
outrageous.
Same deal if you insult someone's intelligence,
same deal if you insult their clothing,
same deal if you just use a bunch of four letter words.
You blankety blank, you know, they won't let me--
I would love to give you a concrete example
but BARBRI won't let me use curse words on the recordings.
But you know what I mean.
So that's a negative example.
In effect, it's a red herring.
In effect, they're trying to sucker you
in it's not outrageous.
On the positive side, there is no conduct
that is definitively and automatically outrageous
but there are certain hallmarks of outrageousness,
certain indicators or markers.
And I'm going to tell you about three of them.
Now, candor compels me to indicate to you
that these are not the only three in the case law.
There are more than three in the case law
but these three are favorites of the examiners.
So I'm going to offer them up as iconic.
The first hallmark of outrageousness
is where the conduct is repetitive in nature.
It happens over and over and over again.
So the conduct has to be bad in the first place.
But then it happens over and over and over again.
In the case law, a good example of this is debt collection.
Somebody calls you up at 3:00 in the morning.
Wakes you up from a sound sleep.
It happens to be somebody who extended you a loan so that you
could purchase an automobile.
You're behind in your payments.
They make a variety of threats.
Their threats, financial threats.
They basically say, if we don't get that payment,
we're going to repossess the car.
We're going to destroy your credit rating,
we're going to put a lien against your house.
We're going to inform your employer.
Your name will be mud.
You'll never be able to borrow money again.
We need that money in 48 hours.
You're extremely distressed.
You can't get back to sleep.
You're having nightmares.
You're unable to concentrate at work the next day.
Does that constitute an extreme and outrageous act by them?
Calling at 3:00 in the morning, making all those threats.
I don't know.
It's probably for the jury. s I'm thinking maybe not enough.
But if they do that every night for a month, calling you
over and over and over again, now it's outrageous.
Harassment.
Perpetual, persistent, unwanted badgering
can become outrageous.
Second hallmark of outrageousness.
This one is weird.
You would never think of this on your own.
I would be surprised if you did.
If the defendant is a common carrier
or an innkeeper, a common carrier or an innkeeper.
Common carriers are transportation company--
United Airlines, Amtrak.
Innkeeper's a hotel.
They are obligated based on principles
that go all the way back before American independence
to English common law, they are obligated
to treat their customers with courtesy and respect.
It's kind of laughable if you have ever been on an airplane.
But it's true and that means that if they deliberately
do something mean, if they deliberately do something
with the goal of distressing you,
then we're going to treat it as outrageous.
You go to check in for your flight,
you go to check in at a hotel.
Person waiting on you says, no, you can't check in.
You're too ugly.
That's outrageous because it's coming
from an airline, a hotel.
That takes me to the final hallmark of outrageousness.
Plaintiff is a member of a fragile class of persons.
Plaintiff is a member of a fragile class.
Three fragile classes to keep in mind here.
Number one, young children.
Young children.
Let's say we were together in the same room.
I see you, I walk up to you, and I say to you, you no good.
Low down, scum-sucking son of a bitch.
You make me sick.
And I turn and I walk away.
You're very distressed.
You're like, the torts professor hates me.
You can't-- you can't concentrate for the rest
of the day.
You skip like the next 10 BARBRI lectures,
you're just freaked out.
Do you have a claim against me?
You do not, that's mere insults.
Mere insults don't count.
But what if I go out on the street and I
see a little five-year-old boy.
I say, hey, come here.
He walks over to me.
I say, you no good, low down, scum-sucking son of a bitch.
You make me sick.
What's his reaction going to be?
Well, he's five, for god's sakes.
He'll start crying and vibrating and pooping all over himself.
But even if he doesn't, it's not about him.
It's about me.
I'm outrageous because I'm violating the principle we all
learned in grade school.
And that principle is pick on someone your own size.
My decision to target a five-year-old is outrageous.
That's the part that's outrageous.
It's not what I said.
It's the fact that I'm targeting a five-year-old.
Outrageous.
Same rule applies for category number two, elderly people.
Elderly people.
Walk down the streets, see an 88-year-old lady.
I say, ma'am, may I speak to you?
Can we have a word?
Lady is like, yes, what is it?
I'm like, you no good, low down, scum-sucking son of a bitch.
You make me sick.
What's her reaction going to be?
Pretty much the same as the five-year-old.
She'll start crying and vibrating and pooping
all over herself, too.
But again, not about her.
It's about me.
Pick on somebody your own size.
Targeting an elderly person considered to be outrageous.
Third and final category here, pregnant women.
Pregnant women.
I'm not making that up, pregnant women,
there are some cases on that.
Yes, indeed.
No hypos.
You can make up your own.
I'm in enough trouble.
Make up your own hypos.
People began asking me a while back.
They used to say to me--
they have been saying to me--
do you have to know that the woman is pregnant?
In order to trigger this sub-rule.
The answer is yeah, yeah, yeah, you
have to know that the woman is pregnant.
You got to use your common sense on all these rules.
What kind of rule would it be if you didn't have to know?
It would be like a trap.
Well, that would make no sense.
What do we-- it's like some secret cause of action
that we would give to women during their first trimester?
They'd go out to a restaurant, server would be rude to them.
They'd be like, got you.
I got my little tort baby in here.
A very lucrative three months.
That would be insane.
Of course, the woman has to be large with child
and then you say something about that.
The examples that come to mind, I will not say.
So let me pause there with those hallmarks of outrageousness.
I'm going to add one more item on this subject
but let me take a little bit of a breather at this point
and we will come back shortly and pursue this some more.
Now, in the topic that we have been exploring,
this idea about extreme and outrageous conduct,
there is one other scenario that is
worth your special attention.
And this one does come up quite a bit on the exam.
If a defendant has prior advanced knowledge
that the plaintiff has some particular emotional
sensitivity and deliberately targets that sensitivity,
that conduct is always treated as outrageous.
You have advanced knowledge--
leave the subject entirely.
The biggest problem with what I just told you
is not being smart enough to figure this out.
It's kind of obvious once it's been called to your attention.
The problem is that in the heat of the moment,
you're reading these questions, you're under time pressure,
the adrenaline is coursing through your body,
and you'll miss it.
So I like to say to folks, bar exam--
50% test of your knowledge of law,
50% test of reading comprehension.
You got to read every word.
You got to pay attention to every word.
If you glide over something, you could
miss the one thing that actually determines the question.
So just bear in mind that one of your goals as you practice
is not just to get the questions right
but to develop the skill of reading every word while
still moving at a steady clip because you
have to watch the clock.
So enough with the sermons.
Let us move forward.
The final three torts on my list all deal with property.
Next one in sequence--
trespass to land.
Trespass to land-- two elements.
Number one,
I want to look at each element with some greater
attention.
Physical invasion can be accomplished
in either of two ways.
The first element can be accomplished
in either of two ways.
First way that you can have a physical invasion
is to simply go on the property.
Defendant enters the land on foot or in a vehicle.
This is common sense.
This is what everyone understands trespass to mean,
including nonlawyers.
You walk on the land.
That's what everybody understands.
Or you drive on it, you ride your bike on it.
That's trespass to land.
There is, however, an exam trick.
It is going to be a trespass--
you will be liable for trespass even
if you are unaware that you have crossed a boundary line
and are on someone else's land without permission.
You do not have to be aware that you are on someone else's land.
The only intent required is that you get to that location
by a deliberate act.
That could sound a little fuzzy.
Let me make it concrete.
John is hiking on a beautiful Saturday
afternoon in a state park.
He's obviously entitled to be there.
He has every legal right to be there.
It's open to the public.
And he paid the admission fee.
He comes to a fork in the hiking trail.
There is no signage, but he chooses to bear right.
After another mile, he has left the park,
and he is now walking on private property owned
by a farmer named Pete.
Farmer Pete's land is adjacent to the state park.
But the thing is, there was no fence or gate or boundary
marker or signage that said, you are now leaving the state park.
It was completely unmarked.
In fact, it was very misleading because the trail just
continued onwards into Pete's land.
If Pete were to sue Dave for trespass,
Pete would be successful.
That is
someone's property rights is not required.
You don't need to know that you've crossed a boundary line.
Now, sometimes I get pushback on this.
I like it when I get pushback because it
means people are thinking.
So people have said to me, well, doesn't that effectively make
this strict liability?
No.
It does not.
Let me tell you why.
There are situations where you could wind up
on somebody's land unintentionally.
Imagine you're walking down a street,
minding your own business in a suburban neighborhood
when you have a heart attack and you
fall onto someone's front lawn.
Now you have invaded their property but not intentionally.
That is not a trespass.
Imagine you're riding a horse.
Your horse is startled by a loud noise.
It panics.
It starts galloping onto Blackacre against your wishes.
You're pulling on the reins--
whoa!
Horse won't stop.
That's not a trespass.
So that's the first kind of physical invasion.
The other kind of physical invasion
is if yo
because no physical invasion.
Yes, the sound quote "went on his land."
But sound is not a tangible thing.
Or you shine bright lights in your neighbor's windows.
That's not a trespass to land.
Now, those kinds of behaviors are actionable.
If they are continuous and oppressive
under the heading of the tort of nuisance, which I promised you
we would take up very near the end of our proceedings
in that mini module that comes next to last.
But they will not constitute trespass.
It's got to be a physical invasion.
Note, however, that the physical invasion
doesn't have to be destructive.
Throwing a rock through someone's land--
through someone's window, excuse me--
is, of course, a destructive act.
And it is a trespass to land.
But if your neighbor has a pond and you pick up
a flat stone on your own property
and you just skim it across the surface
of that neighboring pond just in a whimsical gesture
to see if you remember how to make it skip and it does skip
and then it sinks and it does no harm at all,
that's still a trespass.
So that's element number one.
Two kinds of physical invasion, each one
with a little bit of a trick.
Once you know the trick, mindless, easy, extra question
in your pocket.
Element number two--
I said it has to be land.
It's a lame hook.
It's a lame basis for me to offer you up a particular rule.
What is the particular rule?
It is that land includes the air above the surface
of a piece of property and the soil
beneath the surface of the property
out to a reasonable distance.
Air above, soil below, out to a reasonable distance
is included in the plaintiff's protected interests.
Well, so what if United Airlines flies
an airplane over your backyard?
Trespass?
They're invading the column of air over your property,
but that's beyond a reasonable distance.
That's 10,000, 15,000, 35,000 feet up in the sky.
But contrast that with this kind of a scenario
that could easily come up in a bar problem.
Little boy who lives next door picks up a ball.
He's 12 years old.
He takes that ball and he throws it across your backyard.
Sails through the air, doesn't touch a thing.
Doesn't do any damage.
Doesn't even ruffle the leaves on the trees.
And it lands harmlessly in the street on the far side
of the property.
That is a valid claim of trespass.
You can sue the 12-year-old, and you would win.
Now, to be more complete, you would win
but you wouldn't recover much.
You would only get $1 in nominal damages
because he didn't do any harm.
The amount of interference with your land was trivial,
but technically you would have a valid cause of action.
So that is trespass to land.
And it takes me to the final two torts
on our list, which I am going to take up together.
I'm going to take them up together because they
deal with the exact same subject matter.
Those two torts are called trespass
to chattels and conversion.
Trespass to chattels-- let me just spell that quickly.
C-H-A-T-T-E-L-S trespass to chattels and conversion--
they both deal with an intentional interference with
personal property.
Personal property is all your stuff, literally everything
you own other than buildings and land.
It's the opposite of real estate.
It's all your stuff.
It's all your electronic toys--
your cell phone and your laptop and your tablet and your gaming
console.
And it's all your clothing--
your scarf and your coat and your hat and your sneakers.
And it's all your furniture-- your sofa
and your desk and your bed.
And it's all your motor vehicles-- your car
and your motorcycle and your boat.
It's all your stuff.
It actually includes the currency in your wallet.
Money is personal property, all those Benjamins or Georges,
if you're the more typical bar candidate.
All of that is personal property.
Now, there are two ways that somebody can interfere
with your personal property.
Number one, they can deliberately damage the item.
They can bend it or break it or tear it or rip it
or scratch it.
I can interfere with your laptop by hurling it
against the concrete wall.
Alternatively, second way that somebody
can interfere with your personal property--
they can take it away from you.
They can deprive you of possession, which
is to say they can rob it.
They can steal it.
I can interfere with your laptop by surreptitiously slipping it
into my backpack when you're not looking and tiptoeing away.
So compressing that information into more colloquial,
everyday speech, these two torts are the private civil damage
remedies for vandalism and theft.
They are the private civil damage remedies for vandalism
and that theft of your personal property.
Your only task beyond recognizing
that there is vandalism or theft going on in a question
is distinguishing between these two causes of action.
The distinction is pretty easy.
It goes to the degree of interference.
The degree of interference will determine which cause of action
to use.
If the interference is relatively small,
if it's a slight or minor harm, the proper cause of action
is trespass to chattels.
Small harm, trespass to chattels.
If the harm is more significant, then the proper claim
is conversion.

Elements of negligence-- four traditional elements:


number one, duty.
Plaintiff must show the existence of a duty of care.
Element number two, breach of duty.
Element number three, causation.
We will see much further down the road
two different types of causation.
There's factual cause, there's legal cause.
Legal cause-- better known as proximate cause.
Proximate cause-- very different, very intellectually
distinct from factual cause.
We'll delve into that when the time is right.
And then finally you must prove some kind of actual damage.
Duty, breach, causation, and damage.
Let me say a further word about that list of elements.
I think it's just a useful orientation
here to match things up.
Duty is predominantly about questions of law.
The rules that govern duty are predominantly
found in the cases and in the restatement.
And that means that in terms of material to cover and material
to prepare for the test, duty is the biggest and most
consequential element.
Breach is predominantly about the facts.
You have to have a legal context,
but the breach element is about the factual narrative
that they give.
You find the breach on the printed page of the exam.
It's not really in the law books.
The causation elements are about logic and policy.
They're really not on the page of the exam,
nor are they in the law books.
They're kind of up here in your head.
You got to reason them through.
They're about logical thinking.
Damages, then, takes us pretty much right back
into the facts with a teeny bit of law as icing on that cake.
But there isn't much to worry about,
as we will see when we get there.
So let me talk to you about element number one.
In the law of negligence, that would
be the element of duty of care.
I always like to begin by just kind of restating
the general notion of the law of negligence
because depending on where you are
in your professional career, it may be a few years or quite
a long period since you've thought about this.
Some of you may have taken basic first year torts
and then taken an elective in the field
and then maybe worked in personal injury.
Others of you might not have thought about this
since you were a 1L.
And depending on whether or not this is your first bar exam,
that could have been a long time ago.
So let's just start from a clean slate.
Whenever you go out and act in the world,
there's a chance that you could hurt someone.
This is not a legal proposition.
This is just an observation about the state of humanity.
When you go out and you act in the world-- when you go out
and you choose to engage in some kind of an activity, whatever
that activity may be, one of the collateral consequences
of your action is that you could hurt someone.
This is because we don't live in isolation.
We live in a crowded world that we have
to share with other people.
Whenever you act, there's a chance
that you could hurt other people.
And you can't really get that chance down to zero.
The only way that you could--
I suppose you could get it down to zero.
You know how you could get it down to zero?
You could stop acting.
You could stop engaging in any activities.
You could stay at home motionless in bed
under the covers.
Now there's no chance that you can hurt anyone else.
But it's not a very productive way to lead your life.
We tried that for a year or so, and we discovered
that it's not pleasant.
So it's just an unfortunate sad reality
that when we act in the world, there's
a chance that we could hurt other people.
And because you can't get it down to zero,
the legal system does not demand that you
get that chance of hurting other people down to zero.
Instead, the legal system says to each of us,
as you go about your various activities,
we expect you to make some effort
to lower the chance of hurting others.
We expect you to be a little bit thoughtful or mindful
of the fact that you live in a crowded world.
And we expect you, therefore, in effect
to take other people into account
and modify your behavior accordingly
to lower the chance that you could hurt them.
We don't expect you to get it down to zero
because it's impossible to get it down to zero.
But we do expect you to make some effort in this regard.
That, in a very casual, colloquial way
of expressing it, is what the legal idea
of duty in the law of negligence is all about.
Using more puffed up legal vocabulary,
duty is a legally imposed obligation
to take risk reducing precautions
as you engage in various activities
for the benefit of others.
It's a legally imposed obligation
to take risk reducing precautions
for the benefit of others.
And the consequence of failing to live up to that duty
is that you have to pay money to those that you may hurt.
That's how the duty is enforced.
If you don't live up to it and it results in harm,
you pay for that harm.
You have to compensate your victims.
So that's what we mean by duty, an obligation
to take risk reducing precautions
for the benefit of others.
Then in turn it seems to me raises two immediate
follow up questions.
Question number one, what others are you talking about?
Who am I supposed to think about in designing
my risk reducing precautions?
Who are the beneficiaries of my effort?
Who are the objects of my solicitation?
Who are the individuals who get thoughtful consideration
as I engage in my risk reducing enterprise?
And then second of all, how hard do you want me to work at this?
You said that I need to take some precautions.
I need to engage in risk reducing precautions.
But how much?
You said I don't have to get it down to zero.
Do I have to take a lot of precautions?
A little precautions?
How are you even going to express that?
Those are the two follow on questions
that are testable on the UBE.
Now, I want to take them in turn.
The first question I want to talk about
is, who are the beneficiaries of the risk reducing enterprise
that negligence law obligates us all to undertake?
In other words, to whom do you owe a duty?
The answer that you would get in looking
at almost any textbook on the law of torts
is that you owe a duty to foreseeable victims.
You owe a duty to foreseeable victims.
Now, man, that's easy to write, isn't it?
It's just two words.
True, multiple syllables, but short words.
But I got to tell you-- maybe true confessions,
maybe oversharing here.
When I was a law student, I didn't
know what the hell that meant.
So I owe a duty to foreseeable victims.
Well, who the hell else would get hurt?
It was just very confusing to me.
And as I had the opportunity to think about it over many years,
and as I had to think about it in order
to be helpful to my own students,
I came to realize that perhaps it
was better to think about it in the negative.
If you owe a duty to foreseeable victims, then
you do not owe a duty to unforeseeable victims.
That's just a double negative statement.
Any double negative is the same as the positive statement.
You do not owe a duty of care to unforeseeable victims.
And what does that mean?
Well, once I recast it in that negative or double
negative formula, what I can realize fairly quickly
is that unforeseeable victims always
lose negligence lawsuits.
Why do they always lose?
Because they're not owed of a duty.
If you're not owed a duty, you can't
win because duty is the first element you have to prove.
So unforeseeable victims always lose.
Now all I need to do is define an unforeseeable victim.
And the best way to do that is to build up
from an example, a concrete example.
So here's the concrete example.
There are two guys who were rushing
to catch a railroad train.
And as they arrived on the platform,
they notice the train they would like to take
is beginning to slowly move out from the station.
Thinking that they might still be able to make that train
and avoid having to wait 30 minutes for the next,
they decide to make a running leap onto a little balcony
or platform on the back of the very last car of the train.
Standing nearby and observing this scene
is a railroad employee.
And thinking that he could be of some assistance,
he decides to give one of the men a boost
or a shove up to add to his momentum
as he's about to jump on that little ledge or platform.
And in the course of doing that, he knocks loose a package
that the man was carrying.
Does this sound at all familiar?
I hope so.
These are the facts of Palsgraf versus The Long Island
Railroad.
Palsgraf versus The Long Island Railroad
is often characterized as the most famous negligence
case ever decided, sometimes cited
as the most famous torts case ever decided, certainly
on everyone's top 10 list.
If you didn't read it--
whoa.
I don't know what the hell that was about.
If you didn't read it, OK, I'll tell you what happened.
Railroad employee gives the man a shovel,
knocks loose the package.
But package filled with high powered explosives--
actually, fireworks-- lands forcefully on the tracks
and causes a huge explosion in the railroad station, which
among other things, knocks over this heavy scales,
these weighing machine where people
could put a penny in and get their weight.
These things were really heavy.
And the thing topples over and lands
on this lady, Mrs. Palsgraf, Helen Palsgraf
to be precise, causing her some harm.
And Helen brings a lawsuit.
So that's the case.
Helen Palsgraf-- world's only unforeseeable victim.
What the hell was the deal with this case?
I have always imagined that I could go back in time,
take my time machine, go back to that railroad
platform in the late 1920s, stop the action
with my magic stopwatch, and interview that Long Island
Railroad employee.
It would be a short interview-- one question.
I would just say to him, hey, you're
about to do something clumsy and careless.
You're going to shove a man on a moving train.
You're going to knock loose his package.
What I want to know is, who do you think might get hurt?
And him being from Long Island and me being from New York,
I know how he talks.
So I can hear his answer.
He'd look at me and he'd say, I'm
shoving a guy on a moving train.
And you want to know who I think could get hurt?
Well, the guy I'm shoving is a guy
who could get hurt because he could fall and bang his head.
And I'd say, OK, good.
Who else do you think might get hurt?
And he'd say, well, there's some other people standing
on that back little platform there.
And if I shove him, he could fall against them
and could produce a domino effect.
They could all go falling and bang their heads.
And I'd say, OK, good.
So you're saying the guy you're shoving
and the other people on that platform.
Who else?
He says, well, he's carrying a packet.
I don't know what the hell is in there, but it could be heavy,
and it could fall--
well, I suppose it could fall on my foot.
So ironically, I myself could get hurt.
I'd say, good.
I like that.
Who else could get hurt?
He'd say, that's it.
The guy I'm shoving, the people standing on the platform there,
and maybe me.
And I'd say, well, what about the lady sitting way the hell
down over there?
And he'd say, what lady?
And I'd say, the one by the scales.
And he'd look at me.
And he'd say, no way.
There's nothing I'm doing over here
that could cause her to get hurt over there.
Now, this case bubbles its way through the New York state
court system, goes all the way up
to the highest court of the state.
Court is closely divided.
It's a seven member court.
Court ultimately comes out 4 to 3.
Majority opinion written by Benjamin Cardozo, then a judge
on the New York Court of Appeals, subsequently
a Supreme Court Justice, and thereafter a law school
in New York City.
And Cardozo decides--
I'm convinced, by the way, that Cardozo
could hear that exact same narrative that I made up.
I'm absolutely convinced of it.
Cardozo decides that Helen cannot recover because in his
view, she was outside what he called the zone of danger.
She was outside the zone of danger.
Now, this phrase had not existed in the law
until Cardozo used it in his opinion.
He coined it.
He came up with that.
It was his creation, his innovation.
It is a testament to his intellect.
It is a testament to his stature as a jurist in American history
that that phrase is still in the law 90, going on 100 years
after he wrote the opinion.
So in effect, what he said is, if you're
outside the zone of danger, you're an unforeseeable victim.
You're an unforeseeable victim.
You're not owed a duty of care.
Not owed a duty of care, you lose.
But as clever as the phrase "zone of danger" may be,
I personally don't think you need it.
And I'll tell you why.
You want to know why Helen Palsgraf lost her case?
It's not very complicated.
It's very simple actually.
It's because she was way the hell over there.
When you act like a careless idiot over here,
the people you are likely to hurt are the people near you.
When someone far away gets hurt, you're like, how the hell
did that happen?
And you don't have to pay for it.
So now I think we've beat this question into submission.
Who do you owe a duty to?
You have a duty to the people near you.
If someone far away gets hurt, they're going to lose.
Wow.
Pretty straightforward.
The trick is that near and far will
vary depending on what activity you're undertaking.
You're walking down the street.
You're not watching where you're going because you've
got your nose in your cell phone and you're
sending text messages.
You could therefore hurt other people
by bumping into them, perhaps very forcefully, perhaps
by knocking them down to the pavement
where they might fall and even break a bone.
That's not very nice of you.
Who's in the zone of danger?
Who's near you?
Well, the people on your same side
of the street within a 10 or 15 foot radius
as you move down the street.
It's a constantly changing cast of characters
as you move down the street.
The people on the other side of the street
are outside the zone of danger.
If by some freak they get hurt, you're
not responsible because you don't owe them a duty of care.
Ah, but what if you're driving a car
down the middle of that street?
Now, if you get in an accident, metal and glass
could go flying.
And you would owe a duty to people
on both sides of the street.
All of the pedestrians on both sides of the street
would be within the zone of danger.
They would all be near you, maybe
not in terms of a tape measure but in terms
of the nature of your activity.
To take an absurd example, if you
are juggling nuclear warheads, everyone in the city
is near you because everyone is in the zone of danger
if you screw up.
So near and far, in and out of zone of danger
are elastic concepts that require
you to use some lawyerly skill and analysis.
But that being said, if someone begins the problem on the bar
exam way far away from the defendant,
way out in left field, way the hell over there,
then that person is probably going to lose.
They're outside the zone of danger.
They're not owed a duty of care.
You only a duty of care to those who are near you, with the word
"near" going in heavy quotation marks, the word zone of danger
accompanied by the phrase, "whatever the hell that means."
It's a variable zone.
Now, one of the things about duty in the law of negligence
is that every rule that we encounter,
or virtually every rule that we encounter,
comes bundled with a companion exception.
When you study these rules, you got
to study them along with their companion exceptions.
And the exceptions shouldn't come as a surprise
because every rule has one.
So let me give you immediately the exception
to the Delores principle, the zone of danger exception.
One word-- that exception is rescuers.
Rescuers are going to be able to recover.
They are owed a duty of care even
if they begin the problem at a considerable distance
away, even if they begin the problem way out in left field.
A rescuer may see that something happened
even though they were standing well away from any danger.
They may then come to the center of the action and get hurt.
And the law takes the position that they
are owed a duty of care.
Ironically, it was Cardozo in a different opinion who
established that principle, or at least fortified it,
when all filled up the pithy observation,
danger invites rescue.
So let me see if we can put some of these principles
to work with another problem.
And I will confess at the beginning-- not my problem--
not my problem.
That sounds like some kind of a disavowal of responsibility.
This is not my creative output.
This is from a real case that came up
in the state of New York.
But let me tell you what happened in abbreviated fashion
and get your reaction to it.
Deluxe Corporation owns a high rise apartment building,
a luxury apartment building, in an affluent, prosperous, safe
neighborhood of a major city.
The Deluxe apartment building has,
on the street level, a garage for the use
of tenants and their visitors.
The garage has a door.
In addition to the one that the cars drive in,
it has a pedestrian door that you
can exit and go out from the garage
directly onto the street.
This is a convenience to the tenants who
might want to park their car and then
go shopping in the neighborhood instead of going directly
up to their apartment unit.
The door is designed so that it can only be
opened from inside the garage.
However, the Deluxe apartment management
has failed to keep the door in good repair.
The hinges have gotten rusty.
The door does not slam fully shut.
It remains slightly ajar.
And therefore, people from the street can now open the door
and enter the garage.
One day, Paula is in this neighborhood
coming to visit a friend.
Paula has arrived by subway.
Paula's friend does not live in the Deluxe apartment building.
But Paula has to walk past the Deluxe apartment building
to get to her friend.
As she does that, she is confronted
by a street criminal, a mugger.
And the mugger pulls a gun.
And then looking for a place to complete the crime in privacy,
notices the defective door into the garage.
He motions for Paula to accompany him.
And he takes her in the garage.
And there in the privacy of the garage,
he relieves her of her purse and her watch and runs away.
Paula, badly shaken by the episode and now out several
hundred dollars that were in the purse and out the watch,
cannot sue the mugger because he's never apprehended.
So instead, she sues the Deluxe corporation that
owns the apartment building.
And she points to the negligence of the door--
the negligence maintenance of the door as the grounds
for her lawsuit.
The question I want to know is, did the Deluxe apartment owners
owe her a duty of care?
So yes or no and a one sentence explanation of your thinking.
So as I indicated, this is based on a real case.
So I really don't have to do much work
to tell you the answer.
The court said that the victim here, the plaintiff in the tort
suit, was not owed a duty of care--
not owed a duty of care because not in the zone of danger, not
a foreseeable victim.
What the court said is that the only foreseeable victims
from negligent maintenance of that garage door
were the residence of the building.
They were the ones who are most likely to be
accosted by a criminal as they got out of their own car.
They were the ones whose apartments
might be robbed by a burglar who came into the building
through the defective door.
Passersby on the street were considered
to be too far away and outside the zone of danger.
Now, to the extent that that is a defensible decision,
I think the most important fact that I
gave you was that the building was
located in a safe, low crime neighborhood of the city.
If criminality had been common in the neighborhood,
if it was a crime-infested neighborhood, then
it would have been far more foreseeable
that passersby could be victimized
in this exact fashion.
So what I think really changed the dynamic of the court's
thinking was the fact that there had
been no similar incidents of this sort in the neighborhood.
Now, this is one of those questions where I honestly
will tell you that I'm not sure there is a definitively
and platonically right answer.
If you said, yes, she was a foreseeable victim because she
was in the zone of danger because she was right in front
of the apartment building at the time the crime took place,
I think that's good.
I think that's fine.
I think this would likely be an essay question.
And I think you would get full credit for that analysis.
The question is whether or not you would recognize it
as a zone of danger question and talk about the factors that
involve the defendant's obligation
to anticipate who might get hurt by their carelessness.
So in the real case, in the real facts,
because they could not anticipate harm to passersby,
because street crime was just very rare in that neighborhood,
the only foreseeable victims were the people
who lived in the building.
So that's how they came out.
Let me come back to the narrative of the material.
I indicated to you when we first set up the duty stuff
that if you view duty, as I do, as an obligation to take risk
reducing precautions for the benefit of others,
there are two questions.
I said question number one is, what others are we
supposed to be thinking about?
We've already talked about that.
Question number two, how much effort
am I supposed to make at risk reduction?
How hard am I supposed to work?
That's what I want to tackle now.
The answer to that question is, in my opinion,
the single best known proposition of law
and all of American jurisprudence.
Now, there's a little bit of self-congratulations in that
because I'm a torts guy.
I might have a different candidate
for best known piece of law if I was a civil procedure guy
or if I were a property guy.
But I'm a torts guy.
And the proposition that comes into play
here is that you owe the same amount of care
as would be exercised by a hypothetical reasonably
prudent person acting under similar circumstances--
a hypothetical reasonably prudent person
acting under similar circumstances.
So I said it really slow as if I was trying to allow you time
to write it down.
But here's the thing, guys.
You already know that.
These are notes.
You're taking notes for yourself.
You don't really need to write that down.
You can certainly write it down in the form
of a quick abbreviation.
You know that it's reasonably prudent person.
You don't need to write it down--
you're not going to show this to somebody.
Nobody is going to grade your BARBRI notes.
So write it however you want, or don't write it
at all because you know this.
What I do want to spend a couple of seconds on-- a couple
of minutes on--
is how can they test on it?
The reasonably prudent person standard of care
is notoriously vague.
Not only that, it is the epitome of the kind of question
that we send to the jury.
We round up 12 people, pull them out of their regular jobs,
sit them in the jury box.
And we basically say, you decide if this was reasonable.
So how can they ask questions about this on an exam?
Well, as is my custom, I'm going to take the long way around
to the answer of that question.
In thinking about the reasonably prudent person
standard of care, what you have to bear in mind
is that any standard of care is just a yardstick of comparison.
Any exercise in judging the negligence or fault of a person
involves looking at the defendant
and then looking at the yardstick
and doing a comparison.
Now, the reasonably prudent person standard
is an imaginary standard.
The reasonably prudent person standard of care
involves comparing the defendant to a fictional creature,
a mythical, nonexistent, hypothetical, imaginary
personage.
And because the standard of care is fictional or imaginary,
it has only the attributes that the legal system
has chosen to place upon it.
So what are those attributes?
Let us try to get some sense of what the law means
by a reasonably prudent person.
First thing I can do is begin with a negative.
Reasonably prudent person has no physical attributes
whatsoever--
no height, no weight, no age, no race, no hair color,
no eye color.
You cannot visually picture the reasonably prudent person.
If I say to you right now, close your eyes.
Imagine the reasonably prudent person.
You should get nothing.
Basically, I think of the reasonably prudent person
as the illegitimate child of the Pillsbury
Doughboy and the Michelin Man.
They have a baby.
It's just a big blob.
The reasonably prudent person has no appearance.
What it has instead is a series of character traits.
And the short form version of this is basically,
the reasonably prudent person is somebody
you would never, ever want to go drinking
with because the reasonably prudent person is just
human buzz kill.
This is a person who's just a little bit too attentive,
a little bit to alert, a little bit too persnickety,
and a little bit too careful.
Not exactly a barrel of fun.
Reasonably prudent person is the kind of person
who always locks the car door whenever leaving the car even
if they're just running into the 7-eleven to buy a beverage.
And not only that, the reasonably prudent person
is the kind of person who always carries in the trunk of the car
flares and one of those silvery rescue
blankets and those orange breakdown triangles,
just in case.
The reasonably prudent person is the kind of person
who always arranges when they're about to head out
of town for a week or so to have somebody come over to water
the plants and feed the pets.
Do you do that?
Because if you do that, I give you two thumbs up.
I try to do that.
I really do.
But when I'm going on a trip, I am just a mess.
Just a high anxiety moment for me.
The Uber guy is outside.
He's sending me text messages.
I know that for every four seconds I make him wait,
they're going to charge me more money.
I'm running around the house.
It's like, oh, I didn't pack toiletries.
And oh, I should check if the stove is off.
And oh, is the back door locked?
And I'm kind of in a frenzy.
And I'm like, well, whatever, I'll buy the toothpaste
when I get where I'm going.
And I get the Uber and I fly away
and I do BARBRI lectures in some foreign land--
some foreign land like California.
I go do my BARBRI.
I come home a week later.
And what do I find?
Skeleton of a cat in the kitchen.
It's like, oh, Fluffy.
Not going to be able to call you Fluffy anymore.
Call you Boney.
I forgot to leave you food.
You're not looking so good.
The reasonably prudent person would never do that.
The reasonably prudent person would never ever
spray a lighter fluid on a lit charcoal fire
because the reasonably prudent person would deprive themselves
of that giant thrush of flame that
makes your barbecue the exciting event that it truly is.
In other words, most of us stagger through life well short
of the reasonably prudent person,
thanking our lucky stars that we haven't killed or maimed
anyone.
So that's the reasonably prudent person.
And what we're going to do is we're
going to compare the defendant, the lowly defendant,
the meager, flawed defendant to the reasonably prudent person.
Notice the different location of the hands.
And in making that comparison, we
make no allowances for the defendant's
particular shortcomings.
We make no allowances for the defendant's
particular shortcomings.
The reasonably prudent person standard
is what is known as an objective standard.
It is independent of the attributes
of individual people.
It stands away, outside of individuals.
It is a uniform standard across the entire population.
Everyone in the country is held to the standard.
And it is therefore inflexible and, frankly, kind of harsh.
Consider Dave and Pete are next door neighbors.
Pete has brought a lawsuit against Dave for negligence.
Both houses have detached garages.
Pete proves that Dave left gasoline-soaked rags
in his unventilated garage during the warmest months
of the summer.
Pete shows that those rags then spontaneously caught
fire, burned down Dave's own garage, and then the fire
spread next door and burned down his garage as well.
He is suing to recover money damages to rebuild the garage.
There were no personal injuries here,
fortunately no loss of life, no burn injuries,
just property damage.
Pete proves all of this at trial and then rests.
Defendant Dave gets up and he says,
your honor, I was hoping not to have to say this in open court.
But the truth of the matter is, sir, I'm stupid.
I don't mean that as a figure of speech,
I mean it quite literally, judge.
I have an 85 IQ.
As you know, sir, 100 is average.
I'm substantially below that.
Not too bright.
And judge, I dropped out of school in the 10th grade
because I was finding the work too difficult.
A high school dropout--
that will be me.
And I got to tell you, judge, it never
occurred to me, not in my wildest imagination,
that rags could magically light on fire all by themselves
while sitting in a garage.
I thought you would have to hold a match to them or something.
The idea that they could light on fire by themselves--
I got to tell you, even as we're here today in court,
I am still baffled as to how that happened.
Judge, the plaintiff keeps talking
about spontaneous combustion.
I never heard of spontaneous combustion.
They must do that in the 11th grade.
I'm doing the best I can, your honor.
I'm a decent man.
I try to be a good neighbor, and I meant no harm.
So I'm not liable, am I?
And the judge is going to say, yeah.
You are, because the reasonably prudent person knows all
about spontaneous combustion.
And the reasonably prudent person
would never ever leave gasoline-soaked rags
in a stuffy garage during the warm months of the year.
Now, if that example is not extreme enough
for you, what do we do about somebody who's developmentally
disabled, somebody who's got the intellectual functioning
of a young child?
Are they held to the standard of the average--
excuse me, the reasonably prudent person?
Yes.
They are held to the standard of the reasonably prudent person.
You might say to me, but that's ridiculous.
They can't live up to it.
I'm like, yeah, I know.
Too bad.
What about somebody who has a mental illness or disease,
somebody who's delusional?
They come to court.
I have no time to think about the rags because I am Napoleon.
I must get to the front.
The Russians-- I must fight them.
The judge is going to be like, yeah, OK, Napoleon.
But before you go, pay the plaintiff.
It doesn't matter.
What if you were drunk?
Still doesn't matter.
You have to behave like a reasonably prudent person.
What if you're an amateur or a beginner, a novice?
Doesn't matter.
You got to behave like a reasonably prudent person.
They all have to behave like reasonably prudent people.
That's the point.
So now you know what you're going to see on the exam.
You're going to see a whole bunch of characters
like the kind that I just described.
You're going to see stupid people.
You're going to see uninformed people.
You're going to see uneducated people.
You're going to see people who are inherently clumsy.
You're going to see uncoordinated people, people
who have two left feet.
You're going to see amateurs.
You're going to see beginners.
You're going to see people who are undertaking
an activity that they haven't done for in the past 10 years
and they've forgotten how to do it properly.
You're going to see stoned people.
You're going to see drunk people.
And they're all going to hurt someone.
And there's going to be a choice on the exam.
And it's going to say, they're not liable
if they were doing the best they could.
And you're never ever going to pick that choice
because all of those people have to live up
to the standard of the hypothetical reasonably
prudent person.
Now, when you think about that standard--
that's what you need to know for the exam.
I just have to make this editorial comment.
When you think about that standard, that's
pretty remarkable, isn't it?
What I said to you is pretty remarkable because what
the legal system is doing there is it's basically
asking a significant percentage of the population
to live up to a standard which for them is unattainably high.
And we're not just talking about a scattered few individuals.
Here's a quiz question: what percentage of the population
is below average in intelligence?
It's a trick question.
That would be half.
Average is the midpoint.
So the half of the people who are not as bright as average,
they still have to do as good as the hypothetically reasonably
prudent person.
What percentage of people in the population
are less coordinated than average?
That would be half.
They all have to do as good as the reasonably prudent person.
It's a very hard, harsh standard.
The reasons for this standard, the logic of this standard--
this is something that has been written about
and is fantastic to discuss in a law school classroom.
But it is not a conversation for the present moment.
You just need to understand that it's an objective standard
and that the specific shortcomings of defendant
are not taken into account.
Now, that said, it's a duty rule.
So it comes with exceptions, two exceptions.
Number one, if a defendant has superior skill or knowledge,
then that is taken into account.
The standard then becomes a hypothetical reasonably prudent
person with the same superior skill or knowledge--
hypothetical reasonably prudent person with the same scale
or knowledge.
Doug is a professional race car driver.
He drives cars, competes in NASCAR, very successful,
won a lot of prize money, very celebrated guy.
How is he supposed to drive his passenger
car when he goes out to the supermarket to buy groceries?
The answer is like a reasonably prudent person
with the added skill and knowledge
of a professional race car driver.
If his car goes into a skid, he is
supposed to use everything he knows about controlling
the vehicle in order to prevent that skid in order
to control the vehicle.
He probably knows a lot more about that than you and me.
He's a professional car driver.
Second exception-- where relevant, you
should incorporate the physical characteristics
of the defendant into the hypothetical reasonably prudent
person standard of care.
Where relevant, include defendant's
physical characteristics.
If the defendant is blind, then the standard becomes
a reasonably prudent person who cannot see.
If the defendant has deficits in his hearing capacity,
then the standard is a reasonably prudent person who
is similarly hard of hearing.
If the defendant is 6 foot 8 and if for some reason
another height is relevant in the problem,
then the standard becomes a hypothetical reasonably prudent
6 foot 8 inch tall individual.
Just remember that physical characteristics are often
not relevant.
Blind people, people who are hard of hearing,
and exceptionally tall people should not
store gasoline-soaked rags in stuffy garages
during the month of August.
The ability to see, the ability to hear, and your height
have nothing to do with it.
So don't start thinking about that
unless it is fairly presented in the facts of the problem.
Don't go chasing ghosts.
So that's our basic standard of care.
That the standard of care is what
I call the default standard.
It applies automatically and in every case
unless it is affirmatively displaced by an alternative.
But it turns out there's tons of alternatives.
And so our next order of business
is to look at special standards of care.
I'm going to take you through a total of six special standards
of care.
These are the classic testable standards.
And all of these will make an appearance
at some point on your exam.
So this is ground zero for your last minute review.
First, a special standard of care
applies in negligence claims against children.
First rule, children under the age of five
owe the rest of the world zero standard of care.
Children under the age of five, therefore,
cannot be liable for negligence.
Now, that's not a very important rule.
You can put it in square brackets.
You can put it in a smaller font.
They don't test on it once in a blue moon--
once in a blue moon.
They're not going to give you a lawsuit against a two-year-old.
Everybody has an intuition you can't
hold a two-year-old liable.
Two-year-old's not careless.
A two-year-old's cute.
She tries to put the cat's tail in the electric socket.
It's like, oh, isn't that sweet?
But as soon as the kid turns five,
you want to sue the little rugrat
for every penny he's got.
So let me give you the standard for children between 5 and 18.
Those children are held to the standard
of a hypothetical child of similar age, experience,
and intelligence acting under similar circumstances.
They are held to the standard of a hypothetical child
of similar age, experience, and intelligence acting
under similar circumstances.
Now, the addition of those added words, the references to age,
experience, and intelligence, turn
this standard into what I would say
is almost the exact opposite of the default, plain vanilla
reasonably prudent person standard of care.
The child's standard of care is a subjective standard.
It is extremely flexible.
It is different for every child that you encounter.
Put 100 kids in a room.
Every single kid is going to have
a different standard of care.
Remember, put 100 adults in the room.
Standard is the same for everybody.
Smart ones and dumb ones--
same.
Coordinated ones and clumsy ones--
same.
But you get 100 kids in a room?
All the standards are different.
And the result of this flexibility
and this subjectivity is that the child's standard of care
is a pro defendant standard.
Imagine this: I got two children, next-door neighbors
in an upscale suburban neighborhood,
playing outside their homes.
I'm going to make them five years old.
I'll call one of them Billy and the other Susie,
both five years old.
Billy is riding a bicycle.
It's that scaled down little boy's bike.
He just graduated to it from the tricycle
that he used to have when he was three and four.
He's riding back and forth up and down the block.
His mother said, don't go off the block, Billy.
He's a good boy.
He doesn't go off the block.
He goes down to the corner and makes a U-turn.
Five-year-old Susie is enjoying herself
playing a game of jacks.
She's in a little sundress sitting on the warm pavement.
She bounces the ball.
She scoops up the jacks.
She's kind of engaged in this and not
paying much attention to Billy.
In fact, the two kids are in their own worlds
until Billy comes down the block on his bike and rides
right over Susie's hand and breaks all of her little bitty
fingers.
And Susie is not pleased.
So you know what she does.
She sues Billy's little bitty ass
because she wants to get some cash.
She takes him to court and she says, pay up little Billy.
The question is, what standard of care
was Billy supposed to exercise in operating his bicycle?
Look at your standard.
Number one, he owed the care of a hypothetical child
of the same age.
And I said that he was five, just barely old enough
to be negligent.
Now, if he had been seven, he would have been a hell of a lot
more coordinated.
He would have had better reflexes.
He could have been able to steer better.
He might have been able to swerve and avoid her hand.
But he doesn't have to do as good as a seven-year-old
because he's only five.
And not only that, the standard text into account experience.
And I told you he had just graduated to the bicycle
from his little boy trike.
He was a little wobbly.
He was a little bit uncertain.
And the fact that he is a novice, the fact that he
is a rookie, the fact that he is a bicycle virgin
is taken into account in fixing his standard of care.
And not only that, the standard mentions intelligence.
And there's something I forgot to tell you about little Billy.
He's stupid.
This nickname on the block is stupid Billy.
His parents are up at night worrying about him.
He's flunking out of kindergarten.
They're like, what are we going to do with this kid?
So what's the standard of care?
Let's recap.
How carefully must Billy operate his bicycle on that street?
The answer is he must operate his bicycle as carefully
as a stupid five-year-old who's never been on a bike before.
Susie can only win if he did worse than that.
He has to do worse than that to be negligent.
That's his standards.
Oh, he has to fall below that to be negligent.
Well, good luck.
That's a pretty hard case to me.
That's why I say it's a pro defendant standard of care.
Now, I'm not saying that you can never win negligence claims
against children.
Hop on Westlaw, hop on Lexis-- tons of cases
where plaintiffs have succeeded.
But it's an uphill struggle.
It's a pro defendant standard of care.
And yeah, it's a duty standard.
So you know what that means.
It comes packaged with this exception.
Here's the exception.
If a child is engaged in an adult activity,
then don't use the special standard.
Instead, use the objective, default, plain vanilla
reasonably prudent person standard that we
covered a few minutes ago.
If it's an adult activity, use reasonably prudent person.
What is an adult activity?
On the bar exam, operating a motorized vehicle.
And it's not just a weird bar exam thing.
That's pretty much the case law.
That's the only activity that seems to get litigated.
And that's the only one that is consistently held
across the country to be an adult activity.
Just be vigilant in looking for all different kinds
of motorized vehicles.
Just keep your antenna sensitized.
Yes, of course.
If I have a 15-year-old or a 14-year-old driving a car
or a pickup truck or even a motorbike,
you're going to get that.
But don't forget watercraft, in other words a motor boat.
Don't forget farm equipment--
tractor, combine.
Don't forget things that are only operated for fun--
jet ski, snowmobile.
All of those are motorized vehicles, so all of those
would implicate the adult standard
of care, the plain vanilla standard, the default
standard of care.
So let me take you into the next of my special standards
of care.
If we're keeping count, which we should, it's number two of six,
and this next standard of care is
the standard for negligence claims against professionals.
Negligence claims against professionals
are conventionally called malpractice claims.
That word doesn't really have a legal meaning.
I know you think it may.
It comes up all the time.
People say, oh, I am a malpractice lawyer,
or it's a malpractice case.
Malpractice is just a word that means
you're suing someone who practices a profession.
For some reason or another, you practice law.
You practice medicine.
I don't know why.
You don't practice carpentry.
You don't practice plumbing.
You just do plumbing.
You are a plumber, but you practice law.
So that's why it's malpractice.
I'm not going to define professions.
You'll know them when you see them.
Yes, law, medicine, accounting, engineering, architecture.
But who are we kidding?
98% of the questions will be about medical professionals.
Just a note on medical professionals,
don't get thrown off if they give you some unusual flavor
of medical professional.
In other words, it could be an ordinary medical doctor.
They could specify that it's some kind
of a specialist like a cardiologist
or a dermatologist.
But it could be somebody from some other field of the health
care world.
It could be a podiatrist, a chiropractor, an osteopath.
All of those people are going to be held
to the same standard of care.
So what is the standard?
The standard of care for all professionals
is that they should exercise the same care as an average member
of their profession--
of their same profession providing
similar professional services.
They should exercise the care of an average member
of their profession providing similar professional services.
Now, that is my short form version.
There are more elaborated versions
of this that you can find in the BARBRI materials, more
elaborated versions of this that you
can find in the restatement.
But I think my version is good enough, provided
you home in on the key distinction between the version
I just gave you and the reasonably prudent person.
The key distinction is that I've gotten rid
of the word "reasonable" and I've
swapped in the word "average."
And you might think, aren't those synonyms?
They are not.
By swapping in the word "average,"
I am signaling that the defendant
is no longer going to be compared
to an imaginary standard of care.
Instead, I am signaling that the defendant
is going to be compared to his or her real world colleagues.
The standard of care is now no longer
a mythical or imaginary or hypothetical creature.
It is the professional peer group
of the defendant in the case.
When we do the reasonably prudent person analysis-- when
the jury does the reasonably prudent person
analysis, in thinking about the reasonably prudent person
standard of care, it is not an exaggeration
to say that we're asking them to close their eyes and imagine.
How would you drive during a blizzard?
Imagine.
You would turn on your headlights
and turn on your windshield wipers
and reduce your rate of speed.
And if the snow got really heavy,
you'd pull over to the side of the road
because the reasonably prudent person is imaginary and lives
in your head.
When we ask how you should perform double bypass surgery,
we don't close our eyes and imagine.
We open our eyes and go look.
How do the other doctors do it?
If the reasonably prudent person is objective
and if the child's standard is subjective,
the professional standard of care is empirical.
It is grounded in the real world.
It is neither objective nor subjective.
It simply looks out into the real world.
It is research based.
It is fact based.
Now, most people fall into two groups on this standard of care
in terms of their ability to process it.
There are some people who get it instantly
and some people who are like, I still
don't understand the distinction.
So let me talk to group number two.
Group number one, you got it.
Excellent.
Group number two, I'm going to try
to give you the standard in two other different ways.
Maybe some other verbiage will help.
There is another way that I can express the standard of care
in professional malpractice cases.
And that would be to say that the duty of the defendant
is to be a conformist.
You have to conform to what is generally done by others.
The legal system assumes that in a learned profession where
judges and lawyers and congressmen and state
assemblyman do not have expertise,
that which is generally done by those within the field
is the best available standard, the best available practice.
The best way to do a cesarean section birth
is the way that the other obstetricians do it.
And so you should conform to that practice.
In other words, we don't want you going off and going rogue.
We don't want you using your own weird techniques.
We don't want you saying, I'm going
to do this particular surgery using garden
implements and a blindfold.
Just do it the way the other doctors do it.
Be a conformist.
Another way that we can make the same exact point--
I'm just reiterating the same idea with different wording.
In professional negligence claims,
the custom of the profession sets the standard of care.
That which is customarily done must be done by the defendant.
In ordinary negligence, custom is some evidence
of what reasonable care demands.
But it is not definitively conclusive.
It is not the standard.
It's just evidence of what reasonable people might do.
In professional negligence claims, custom is the standard.
So whether you prefer the formula
that you must behave as the average member
of the profession, or whether you prefer the idea
that you must conform to what is generally done,
or whether you prefer the idea that we must research
the professional custom and hold that as a standard of care,
they are all driving at the same point.
Now, this implicates a further piece of information
that I think you need to have.
The jury is not going to know in advance what
the professional custom is.
The jury does not know, generally,
how medical procedure should be done.
The jury does not know whether you
should treat a particular cancerous tumor with radiation
versus chemo and which chemotherapy
drugs you should use.
The jury does not know how a lawyer should litigate
an antitrust case and whether or not
you should take this kind of discovery
or make that kind of a motion.
The jury has no expertise, and the jury has no familiarity.
So plaintiff almost always needs an expert witness.
The expert witnesses there to, in effect,
tell the jury what is customarily done,
not speaking so much from his own firsthand expertise,
but speaking from his familiarity
with his professional colleagues.
Are you familiar with how this is normally done?
Yes, I am.
Tell us how it's normally done.
Now, this led in earlier days to a controversy about where
the expert had to come from.
Does the expert have to come from the same city, town,
or community as the defendant?
Does he have to come from a similarly sized community?
And the states were kind of all over on that.
But in the last 20 years or so, there
has been an emerging consensus.
So now it is safe to say the standard is
a national standard.
If you're suing a doctor from downstate Illinois, relatively
rural and small town part of the state,
you can get your expert witness from Chicago.
It is assumed that the standard of care will be uniform.
Now, if you're suing somebody who is a specialist,
the expert does have to come from that same specialty.
If you're suing someone who deals with treating cancer
patients, someone known as an oncologist,
then your expert has to be an oncologist.
So let me close with just a reinforcing exhortation,
a sort of emphasis to you.
If you get an essay question about malpractice,
which is not a rare thing on the MBE,
and if you are setting out the standard of care,
the one thing I would really ask you to do
is please do not say--
please do not say that the defendant must
behave as a reasonable doctor.
Write that down.
Do not say, reasonable doctor.
And let me tell you why.
The word "reasonable" is not just a word.
It's a legal term of art.
And you know what it means?
It means imaginary.
So if you write that down, what you just wrote down
is that the defendant should behave as an imaginary doctor.
And that's not correct.
The defendant should not behave as an imaginary doctor.
The defendant should behave the way her colleagues behave.
You're just going to confuse the grader, raise a doubt,
and ultimately impact your score in a negative way.
So I would not use the word "reasonable."
Try to keep that out of the essay.
It just will undermine your attempt
to demonstrate that what's going on.
So enough said about that.
I think that's a good place for us to leave it be.
Module 2A: Negligence Duty and Breach
o 50% of MPT questions will be on Negligence
o Out of the 27 questions you should there will be 14 Negligence questions
o 7-8 on Intentional Torts Questions
o 4-5 questions on all the other topics

Tort Elements of the Prima Facie Case of Negligence


 There are 4 Elements to the Tort of Negligence:

 1) The defendant owes a duty of care to conform to a specific standard of conduct:

 The plaintiff must show that the defendant owed a duty and then specify the
nature of that duty
 You owe a duty of care to all foreseeable victims (people who are near you),
 You don’t owe a duty to unforeseeable victims (people who are far away).
 Zone of danger use common since if it’s a nuclear bomb it’s the whole
world if it’s chopping down a tree it has to be the height of the tree.

 Palsgraf V Long Island Railroad Co. Case


 Two men attempted to board the train, one of the men was aided by railroad
employees who then dropped a package that exploded and caused a large coin-
operated scale to hit Ms. Palsgraf causing her injury. Ms. Palsgraf sued the
railroad, arguing that its employees had been negligent while assisting the man,
and that she had been harmed by their negligence. Her claim failed because she
was not a foreseeable victim and no liability to unforeseeable victims.

 General rule is Judge Cardozo View (Majority)


o Foreseeable Zone of Danger:
 He said Ms. Palsgraf was not owed a duty of care because she was outside the
zone of danger
 People around/near you are the people who could get hurt

 Judge Andrews View (Minority)


o Everyone is Foreseeable:
 Defendant owes a duty of care to the world at large regardless of whether the
plaintiff was a foreseeable victim

 The Exceptions to the Palsgraf Case is


 A rescuer can be out of the zone of danger but comes to the center of the action to
give aid/comfort/assistance to someone who is more directly injured
 If the rescuers in the process sustains injury he will be able to sue in Negligence
 They are not subject to the zone of duty principle

 Wagner v. International Railway Co. Case


 Cardozo stated “Danger invites rescue” so we will treat these people
as foreseeable victims even if out of the zone of danger
 2) The defendant breached that duty
 The plaintiff has to show that the defendant failed to live up to the duty of care by
a specific way by an evidentiary showing

 3) The breach of duty was the actual & proximate cause of the plaintiff’s injury
 Causation comes in 2 different tests such as factual cause and proximate cause

 4) The plaintiff suffered damages to person or property


 You have to show that something happened to you

Practice Question:
 The DeLux corporation owns and manages an apartment building in an upscale and generally
safe neighbourhood of a large city. There is a garage that is part of the apartment building and a
door that leads from the garage out to the street. The door is designed to be opened only from
the inside so you can exit out to the street but it is not supposed to be opened from the street.
However, they unreasonably failed to maintain the door. As a result, the door was able to be
opened from the street side. One evening Patricia, who does not live in the building is walk
down the street when she in confronted by a mugger. The mugger not wanting to commit a
crime in plain view on the street, tries the door and is able to open it. He drags Patricia through
the broken door and into the DeLux Apartment garage where he assaults her, takes her purse
and then flees. Patricia sues DeLux for negligence, citing their poor maintenance of the garbage
door.
 Is she likely to win? Why or why not?
o Patricia will lose because she is an unforeseeable victim and is not owed a duty of care
as she was just a passerby on the street.
o The foreseeable victims would be the residents of the apartment building, who might
sustain criminal attacks once in the building

Standards of Care:???
Subjective Standard of Care
 Must exercise the same degree of precaution as would have been exercised by a hypothetical
reasonable prudent person acting under similar circumstances
 Reasonably prudent person has no height, age, weight, sex, race and no physical attributes
 A reasonably prudent person has a set of behaviors, they are always alert, careful and always
taking advance precaution. Most people to live up to this standard.

Objective Standard of Care


 We make no allowances for the defendants particular short coming, everyone must live up to
the standard of an objective standard
 Inflexible, ridged and harsh standard

 Example 1: Pete claims that Dave kept gasoline-soaked rags in his


unventilated garage during the hottest months of the summer. The
rags caught fire and burned Dave’s own garage and the fire spread
next door and burned Pete’s garage as well. They are both
detached garages. No personal injury or damage to the houses

 Pete sues Dave in a Negligence claim for money to the burned garage, Dave tells the
court he is stupid and below IQ levels. Is Dave still liable?

o Dave is still liable because he has to do as good as the reasonable prudent person & he didn’t
o If the defendant is mentally disabled and has the mental age of a child of 6-7 years old, that
person has to do as good as the reasonable prudent person
o Beginner who engages in activity for 1st time, still do as good as reasonable prudent person

2 Exceptions to the Reasonable Prudent Person Standard of Care:

 1) If a defendant has superior skill or knowledge:


 The standard of care includes a superior skill or knowledge of a hypothetic person
with that same superior skill or knowledge
 Act like a reasonable prudent person with that extra knowledge, as he has superior
knowledge like a professional race car driver
 The Reasonably Prudent Person Standard of care will go higher for professional
knowledge but it will never go lower for the stupid clumsy people

 2) Where relevant defendants’ physical characteristics are taken into the standard of
care (average mental ability but the same physical characteristics as the defendant)
 Where relevant defendants’ physical characteristics taken into the standard of care
 If the defendant is blind, the standard of care will be of a reasonably prudent blind
person, it is not expected for a blind person to see things as they don’t have vision
 We expect a reasonably prudent blind person to take precautions appropriate to
visual impairment, we expect them not to go out without a cane, guide animal or
some assistant device
 Physical characteristics are not always relevant
 Objective, default standard of care that applies in every case automatically by
operation of law
6 Special Standards on Negligence Claims:

 1) Children over the age of 5 must conform to standard of care of a child of like age,
education, intelligence, & experience of a child in similar circumstances

 Exceptions: If a child is engaging in an adult activity the reasonably prudent person


standard of care applies (Adult activity is operating a motorized vehicle)
 Children under the age of 5 owe a zero duty of care
 Every child is subject to a different standard of care, it is a subjective standard so
it varies from child to child and that makes it flexible and lenient
 Pro defendant standard of care, it is hard to win negligence cases against children
 For liability we look at a child of similar age and experience

 2) Professionals must exercise knowledge & skill of member of the profession in good standing
 A standard of care of the average member of the same profession, providing
similar professional services
 Lawyers, accountants, architects, engineers, health care providers such as doctors,
dentists, chiropractors
 A plaintiff in a mal practice case always needs an expert witness, which can be
from any state and not only from the same state because how are we going to
know what is customarily done
 Doctors owe a duty to disclose risk of treatment to patients, prior to embarking
upon that treatment. (informed consent doctrine).
 If doctor withholds that information or fails to share it and if the risk materializes,
you will have a claim in negligence

 When getting a professional negligence questions don’t use the word reasonable it is not reasonable it
is average member
NY Bar - Video 2 -Tort Law
Module 2B: Negligence- Duty & Breach

3) Premises Liability Cases:


 Unknown Tresspasser-Someone enters a piece of property and while they are on that
piece of property they encounter a dangerous condition.- unforeseeable victim no
obligation to deal with hazards
 The question will always be what duty of care does the person in possession of the
property owe to the entrant on that property
 The types of piece of property can be a piece of outdoor open land, a tract of land in a
forest that is being held for future development, a field/orchid owned by a farmer, or a
vacant lot in a city, or an outdoor developed parcel like an amusement park or building,
office building or a commercial establishment like a supermarket or a private home
 Ranges of hazards that someone may encounter entering property can be a chair in your
doctors waiting room, chandelier that is not properly bolted in the ceiling, Icey front steps
 The duty of care is different based on the kind of entrant we are dealing with

A Question on Premises Liability

 Step 1: If it’s a premises liability question, you ask did someone come on land?
 Step 2: What kind of entrance am I dealing with?
 Step 3: Then use the 4- or 2-part test?
 Step 4: Apply the test to the hazard that hurt the plaintiff

Practice Question
 Darryl owns a large ranch. Over a deep canyon there is a wooden footbridge. He recently
had it inspected by an engineer. The engineer told him that, even though it appeared safe,
the footbridge had rotted so that it could no longer support the weight of a person. One day
while out horseback riding he sees Pete, a hiker on his land without permission, walking on
his land towards the bridge.

 Does Darryl owe Pete any duty to protect him from getting hurt on the bridge? Why or
why not? (apply the facts to the 4-part test)
o There is a duty because there is a known trespasser, Pete did not enter with permission
but Darryl has firsthand awareness of his presence
o The duty that Darryl owes a duty to Pete to (4-part test) artificial, highly dangerous,
concealed and prior knowledge
Standard of Care to Landowners:
 A landowner’s standard of care under traditional rules usually depends on the status of the
person injured on the property

1) Unknown Trespassers
 The landowner owes no duty to undiscovered trespassers
 An unknown trespasser always loses a negligence claim in a premise liability case,
you can’t win if you can’t establish a duty and no duty is owed so you will lose
 An unknown trespasser is an unforeseeable victim

2) Known Trespassers or Anticapated Tresspasswe 2


 For discovered & anticipated trespassers, the landowner owes a duty to warn of or
make safe known highly dangerous artificial conditions if not obvious to the trespasser
 Anticipated trespassers are those in which you should expect to trespass even if you
don’t have actual and concreate awareness that they are on the property

 On the exam-they are anticipated trespassers when there has been a pattern of trespassing in the
past because if they people have done it in the past they will continue, so expect them

The duty is possessor must protect only against those hazards on land that
meet 4-part test:

1) An Artificial Condition on the land in Order to Trigger a Duty


 Artificial means constructed by human beings and not natural
 A possessory owes known of anticipated trespasser no duty to natural occurring conditions

2) The Condition Must be Highly Dangerous


 Highly dangerous means capable of inflicting severe bodily injury or death
 No duty to moderately dangerous conditions

3) The Condition Must be Concealed from the Trespasser


 The trespasser must have no advance knowledge and it has to be a hidden condition
 No duty of care in respect to open and obvious conditions, as the trespasser can see
it he can take the necessary precautions he/she might wish to take

4) Possessory of the Property Must have Prior Knowledge that the Condition Exists
 If you don’t know about it you owe no duty

 The duty is to protect only from non-man-made death


traps on the land
3) Licensees

 Licensees are those who come onto the land with express or implied permission/ consent
but for their own purpose

 They do not confer any economic benefit on the possessory of the property but they have
either an express or implied permission
 Most common type is a social guest, a friend you invite over for dinner
 Someone that comes to your door like a politician seeking your vote or girl scouts selling
cookies, religious people trying to convert you

Duty is Defined in a 2 Part Test, Only Need to Protect Those That Are:

1) Concealed from Them


2) Known in Advance by defendant

 The landowner’s duty is the same as for discovered trespassers except that it applies to all
known dangerous artificial and natural conditions

4) Invitees

 Invitees are entering as members of public or purpose connected to the business of landowner

 Invitees enter with permission & they enter to confer economic benefit on the possessory
 You are also an invitee if you enter property that is open to everyone, open to the general public
 A business customer is an invitee such as when you go to the supermarket, hair salon, or airport to
pick up a friend without doing any commerce because the airport is open to the public at large or
enter a museum that is open to the public that does not charge in admission, or a house of worship

 The landowner’s duty is the same as for licensees but with the additional duty to reasonably
inspect for dangerous conditions

2 Part Test, The Duty Arises Only When the Condition is:

1) Concealed from the Invitee

2) Condition Must be Either Known by the Possessor or Could be Discovered through a


reasonable inspection

 A reasonable inspection is conducted by a reasonably prudent person. It is thorough and takes


cost into account and occurs at reasonable intervals, it does not require to be inspected every
day or every hour
 There will still be some hazards that could not be discovered through a thorough inspection
because they are so well hidden, to those you owe no duty
 Note that the ordinary reasonable care standard applies for active operations on the property
and for conditions on the land that injure children (the “attractive nuisance” doctrine)

Premises Liability Scenarios


1) Firefighters and Police Officers
 Firefighters and police officers frequently come on land, it is in the nature of their
job to enter property
 They are never allowed to recover for injuries that are inherent risks of the job
because they have the theory of assumption of the risk as they know what they are
getting into
 What are the inherent risks of a firefighters?
o Getting burned, smoke inhalation, getting hit by falling pieces of drywall or
timber as you enter that burning building
o If this happens you don’t categorize him as a kind of entrances or go through a 2-
4-part test
 What are NOT Inherent Risks of Police Officers Jobs?
o They come to a house for a domestic dispute and sit on the chair and it collapses
o It has to be linked up with the work

2) Trespassing Kids (attractive nuisance doctrine)

 A property owner must exercise reasonable prudent care to prevent injuries to


trespassing children from artificial conditions on the land
 Assess the likely hood that children will come on the land
 If the land is in the middle of nowhere and a neighbour is 10 miles away, the odds of
trespassing children a not likely, it would be reasonable to leave hazards on the land
 If the land is across the street from a middle school and a swing set is visible in his
backyard, then assessing the likelihood of trespass is very high
 The swing set acts as a child magnet, an assessing the likelihood of trespassing children
ask yourself if there is anything any item that is visible in your land that will draw
children
 If there is an item that is a child magnet then reasonable prudent dictates that he should
make some effort to make the property safe

3) If we have a Duty that is Owed in Any Scenario Other than Trespassing Children
 If a duty that is owed to an adult, trespassing adult or licensee or invitee

 There are 2 ways in which you can satisfy your duty:

 1) Fix the Hazardous Conditions, to Eliminate the Danger


 To repair or make the condition safe
 If you have ice on your drive way put salt to protect guest from slipping
and falling on the snow
 If you have an unstable chair fix it or throw out that chair

 2) Give a Warning
 Instead of fixing the item itself you can provide information
 You can give either an oral warning or you can use a sign
 You can tell your guests don’t sit on that chair it’s kind of flimsy
 You can put up a sign do not cross this bridge, it is icy and put a chain

4) Statutory Standards of Care (Negligence- Per Se)


 In the law of Negligence plaintiffs will often search for a criminal Statute that covers the
conduct that the defendant engaged in and ask the judge to use that statue as a one time
this case standard of care, by shutting down and eliminating any possibility of excuse
 There is a criminal statute that say you can’t run a red light and if you do you will
be subject to a fine and points on your license
 If you find that he ran the red light, you must find that he was negligent

When Can You Use a Criminal Statute in Civil Litigation to Prove the
Standard of Care? (2-part test of class of person class of risk test)

 1) Is the Statute designed to protect a class of people to which this plaintiff


belongs to or is a member of

 2) Must prove that the statute is to prevent the type of risk that materialized in
this case. The injury that occurred is the kind the statute is trying to prevent

 If the test is satisfied then you can use the statute


 If you can prove a statutory violation, that statutory violation is conclusive of a breach
and no excuses are allowed
 This is called Negligence- Per Se

Practice Question:
 Delores goes to work unware of a slow gas leak from the stove in her kitchen. That evening,
when she returns to her apartment, she is very stressed, so to relax she decided she would like
to smoke some marijuana. When she lights the joint there is an explosive from the gas leak
which blows out the wall of the apartment and damages the furniture of Delores’s neighbor,
Patricia. Marijuana is illegal in this state, punishable by 3 months of jail time. Patricia sues
Delores for negligence.
 Can Patricia use the criminal statute regarding marijuana use as the standard of care?
Why or why not?
o Patricia will not be able to use this statute because she can’t establish the 2-part
prong of class of person class of risk
o The statute that criminalizes marijuana is designed to protect against health risks not
from blowing up your apartment
o The statute is designed to protect the marijuana user from her own decision of using
the drug, it is not designed to protect next door neighbors
 Therefore, it fails the 2-part test and you cannot use the statute but she can
use the prudent reasonable person standard of care

 What does a reasonably prudent person do when they walk into a house with a gas leak?
o They usually would smell it and call the fire department
o A reasonably prudent person would not smoke marijuana or use any fire when there
is a smell of gas

Exceptions to the Statutory Standards of Care on Negligence- Per Se

 1) Even when the 2-part test is met, if statutory compliance/follow would have been
more dangerous than the statutory violation, then don’t borrow the statute

 Example1: Dave is driving on a road that has a yellow line that means
you cannot pass to the other side. A child then runs onto the
road and Dave swerves over the yellow line to avoid hitting
the child. Pete a bicyclist sees Dave swerving in the other
lane and he the swerves and hits a tree. Pete is badly hurt
and sues Dave. Pete wants to borrow the statute that says
you do not cross a double yellow line. Class of person class
of risk is satisfied here, it’s a traffic ordinance protecting
motorists from head on collisions. Pete will not be able to
use the statute because if Dave didn’t go over the yellow
line he would have hit the child. Dave violated the statute in
order to mitigate or avoid much more serious harm. This
will be analyzed under a reasonable prudent person under
the circumstances.

 2) If statutory compliance was impossible, don’t use statute even if the 2-part test is
met
 Example2: Dave a motorist has a heart attack and hits a pedestrian.
Motorists runs a red light, so he will use the red-light statue
and class of person class of risk is satisfied. But if you have
a heart attack and can’t control your vehicle, statutory
compliance is impossible. Dave can’t stop the car because
he is incapacitated. Here we fall back on a reasonable
prudent person standard of care.
 Plaintiff wants to know if Dave had chest pains prior to the heart attack because if he was
a reasonable prudent person he would have pulled over to the side of the road and dial
911 on his cell phone or
 If he is supposed to be taking heart medication and missed it this morning, as this would
be an unreasonable thing to do
 The violation of the statute will not be the standard of care
5) Duties to Act Affirmatively (Special Standards/duty in the Law of Negligence):

 There are no duties to act affirmatively by taking a course of conduct, if you don’t want
to. If you don’t want to drive & take a bus then you don’t have to drive & take a bus, no
one will make you. However, once you decide to drive you have to drive as a reasonably
prudent person would, including taking affirmative steps to drive as a reasonably prudent
person
 You have no duty to act to rescue a person in peril, evening if a child is drowning

 Exceptions 2 Scenarios Where There is a Duty to Rescue:

 1) If there is a pre-existing relationship between plaintiff & defendant,


that triggers a duty
 Employer-employee, land owner-invitee, innkeeper-guest, common
career-passenger

 2) If the defendant is the one who caused the peril or put the plaintiff in
peril, that triggers a duty

 The duty in both of these scenarios, are a duty to act reasonably under these circumstances
 You are never obligated to put your own life in jeopardy and rescue may not be feasible
under the circumstances
 If you opt to rescue and you perform the rescue negligently, you are liable
 Many states changed that rule with a statute called good Samaritan laws

6. Negligent Infliction of Emotional Distress (Special Standards/Duty Situations)


 General basis of liability: The defendant breaches a duty to the plaintiff by creating a risk
of physical injury and the plaintiff suffers emotional distress as a result

There are 2 Requirements to Recover in a Near Miss Case:


 1) The plaintiff must be within the “zone of physical danger” and
 2) As a result of the emotional distress you then suffered subsequent physical
manifestations

Exceptions:
 The negligent defendant breaches a duty to a bystander not in the zone of danger who
 (I) Is closely related to the injured person,
 (ii) Was present at the scene of the injury, and
 (iii) Personally, observed or perceived the event
Business Relationship Cases
 Involve a plaintiff and a defendant who are in a pre-existing business relationship, where
careless performance is highly likely to cause emotional distress

 1. Medical Patient & Medical Laboratory


 Example 1: P goes to the doctor because she has a strange growth in her
arm. The doctor takes a sample and sends it to the lab. The
lab negligently performs their work and informs P that she
has cancer.
 Is P Emotionally Distress? Is this Foreseeable?
o Yes, she is emotionally distress, if you are a lab testing for cancer you know that
screwing up will have this consequence
o This is a good case for a business relationship of negligent infliction of
emotional distress

 2. Funeral Parlor & Patron


 Example 2: P goes to a funeral establishment, she is already emotionally
fragile. She is making arrangement for a family member.
 Is it foreseeable that if they screw up by cremating instead of a casket or they lost the
body, will P be more distress?
o Yes, this is a good claim for negligent infliction of emotional distress
 3. Dry Cleaner & Customer
 Example 3: P brings in her favourite shirt to the dry cleaner. 3 days later
she comes to pick up the shirt and the dry cleaner has
destroyed the shirt. She is screaming in emotional distress,
however in this case there is no business relationship of
negligent infliction of emotional distress.
 Is it not foreseeable that screwing up the dry cleaner will cause emotional distress?
 It is not a relationship predicated on that

Practice Question:
 Pamela was sexually harassed by her manager at work. She managed to surreptitiously
record some video of the episode on her cell phone. The next day she asked a co-worker
named Carl to held her file a complaint with the EEOC, and she shared the video with
him to show what had happened. Carl helps her file a complaint. When management
learned that Carl had gotten involved they fired him, and Carl then decides to sue the
company for retaliation. He retained Danielle as his lawyer. Carl turns over to Danielle
the video that Pamela had given to him. The case received a lot of publicity and Danielle
gave some interviews about it on local TV. In one of those she showed Pamela’s video.
Pamela was very humiliated and emotionally upset when the video aired, and she has
sued Danielle for negligent infliction of emotional distress.
o Is she likely to win? Why or why not?
o The court held that there was no cause of action because there was no direct
relationship between Danielle the lawyer and Pamela the plaintiff
o This is a business relationship case, however here Danielle was Carls lawyer not
for Pamala
o No business relationship
o If no physical injury, then it has got to be a relationship case

Module 2C: Negligence- Breach of Duty

 Plaintiff must demonstrate exactly what the defendant did wrong and reasons why it is wrong
 Explain why this conduct falls short of the standard
 You need fact plus reason in order to make a breach showing
 Plaintiff will claim this is unreasonable because (reason-) alcohol impairs your ability to drive
 A behaviour might be short of the standard of reasonable prudence its costs outweighs its benefits

Res Ipsa Loquitur

o Res Ipsa Loquitur is a doctrine used by plaintiffs who lack information about what the
defendant did wrong

o If you have not specified a particular breach, then how do you know you sued the right person?

o Must establish that the likely hood that you sued the right person by showing that the
defendant had exclusive control over the injury causing item at the relevant time

 Byrne V Boadie 1863 Case from England. Mr. Byrne (Plaintiff) testified that he
was walking along Scotland Road when he passes a bakery and then evidently lost
consciousness by a barrel falling and hitting him on the head. As Mr. Byrne lost
consciousness he did not know who or how it hit him but he did sustain injuries.
Mr. Byrne brought a negligence claim against Mr. Boadle (Defendant) the owner of
the Bakery shop. The defendant’s shop was adjacent to the road on which Plaintiff
was walking, and the barrel appeared to have fallen, or was dropped from the shop.
In a regular case this would have been dismissed because no evident of a breach as
no indication to the breach.
 A plaintiff must persuade a jury that more likely than not the harm-causing event
does not occur in the absence of negligence.
 As the barrel was in exclusive control of the bakery then whatever screw up
occurred it is on them

o Res Ipsa Loquitur is a way that you can avoid a directed verdict by making 2
substitutes showing instead of direct evidence of a breach

2 Substitute Showings Instead of Direct Evidence of a Breach

1. You have to show that the accident is of a type/category which is normally associated
with negligence

2. You have to show that an accident of this type would normally do to the negligence of
someone in the defendant possession

 When you show these 2 things you get to the jury, as it is enough to satisfy the
prima facie case of breach.
 The jury can still come up with a verdict either way as there is no guarantee of a
victory just because you made a claim using Res Ipsa

Module 3A: Negligence/Causation-Factual & Proximate Causation


 On an essay question always do factual causation first(alphabetical F comes before P)
 The jury decides causation
 When multiple defendants or merged causes don’t use But for Test but substantial test

Factual Causation

o This element is the point in the case where the plaintiff establishes a linkage or
connection between the breach and the injury suffered

 How do you show the connection/link between the 2?


o But for Test- (Latin for but for- sine qua non)
o Whether the breach was essential/necessary in producing the injury

 How does a defendant counter argue the But for Test?


 The argument usually starts like this:
o Even if I have been careful, you still would have gotten hurt

Multiple Defendants (2 merged causes)

 A classic example is 2 negligently set fires


 Example 1: Dave is smoking and throws his fag out the window, this is
not a reasonably prudent thing to do. The fag lands in a bed
of dry leaves and sets a fire (fire A). At the same time but a
mile away, Doug is camping in the woods and packs to go
home but forgets to extinguish his camp fire (fire B). The
wind picks up and the two fires join/merge and burn down
Pat house (plaintiff). Pat is now suing both Doug and Dave.
 In this kind of case we have a merged cause and the proper test in causation would be
the Substantial Factor Test

Substantial Factor Test (merged causes)

 In a substantial factor test analysis, you ask whether each breach contributed to the injury
in a significant or substantial way
 If a breach, had it been the only breach in the story would it have been capable of causing
all the harm by itself then it is defiantly a substantial factor.

 If a breach had 2 substantial factors, you hold both of them jointly in the fire hypo
(example1) either fire in existence would have caused the plaintiffs house to burn,
therefore both the fag tossing(fire-A) and leaving the camp fire on (fire-B)
carelessness, both would be treated as substantial factors. Both breaches should be
therefore considered factual causes. If you hold both of them being substantial
factors you hold both of them jointly liable
Alternative Causes Approach Test: Multiple Defendants

 Multiple defendants and only 1 true cause of harm but unascertainable cause case
 The plaintiff must establish his case by a preponderance (quality of facts) on every single
element
 Shifts the burden of proof to defendants
 Each defendant must show their negligence is not the actual cause “it was not me
because...”
 If neither can prove its not them, then you hold them both jointly liable

 Summers V Tice 1948 case- 3 guys(Moe, Larry, Curly) went hunting with shot guns
for quail. Quail are ground birds that you have to get
them out of their hiding by getting a stick and shaking
the bush and they fly out into the sky (beating the
bushes). When they fly out you shoot them with 1 pellet
and kill it you don’t kill it with a bunch of pellets
because it will vaporize the bird. As the birds are flying
in the air both Larry and Curly shoot at the same time but
missing the birds. However, 1 pellet hit Moe in the eye
leaving him blind in that eye. Moe then sues both Larry
and Curly for negligence. The issue here is that only 1
pellet blinded him, so only 1-person carelessness caused
his injury and the others did nothing, but the plaintiff
does not know which one. The courts shifted the burden
of proof so the defendants can talk their way out of the
case by proving that neither of them shot him.

Sample MBE Question on Multiple Defendants

 Pete was in a car driving on a country road and there were 2 farms on either side of the road.
One farm was owned by farmer Dave and the other was owned by famer Dough. Each
farmer owned a chestnut brown horse. Farmer Dave negligently let the fences that separated
his pasture from the road fallen in disrepair. Farmer Dough negligently left the gate open
that separated his pasture from the road. As Pete was driving on the country road he sees a
horse, to avoid hitting the horse he swerve out of the way and hits a tree. The horse gallops
away. Pete sue both Doug and Dave for their Negligence because he doesn’t know which
one actually caused it.

 There was only 1 horse on the road,


 Only 1 breach that could have caused the harm, but we don’t know which one,
 It is unascertainable
 It is up to each defendant to talk there way out of it if they can
Proximate Causation (fairness in liability)

 The plaintiff must convince that liability would be fair


 Proximate cause (fairness) will be judged based on whether what happened was a foreseeable
consequence of this particular breach
 It is fair for people to pay for their foreseeable consequence, but not fair to pay for
unforeseeable consequence
 First question you ask is if it is an unusual, freakish or bizarre case, then look at the breach,
catalogue the risk that you would have expected before hand and then check the risk that
materialized. That is a foreseeability analysis and you do that in the spirit of fairness
 If fairness is self-evident/obvious you do not need to demonstrate
 Example1: Paul was waiting at the tube station to ask a TFL worker
directions. The worker was helping a Dan get on the tube and
shoved him in. Dan as he was getting shoved in dropped a
package and it burned Paul. Paulo was a foreseeable victim,
however what had happened was that Paul got burned.
 When asking if someone is foreseeable you have to think of what risks can come out
of that breach or what is associated with the breach.
 The breach is shoving and knocking a package

 What is associated with the breach and what risks come from that breach?
o People can fall- if they fall they can get a bruise or a broken bone
o Knocking a package down can cause the content to break
o Knocking a package down can cause the package to fall on someone’s foot and
cause an injury in a form of a broken foot
 These are all foreseeable risks BUT
 Shoving and knocking a box do not foreseeably cause explosions and fires.
 That was a big surprise and because it was a big surprise TFL won’t be liable.
 It would be unfair to make TFL liable because it played out in such a
surprising way, they couldn’t have known the box would burn someone.
 This was an unusual, freakish or bizarre case.

4 Precedent in Proximate Cause (important)


 Courts have determined that all these outcomes are foreseeable, liability is fair, that the
defendant should be treated as a proximate cause and the plaintiff wins.

Original plaintiff will always be liable in cases involving:

 1) Intervening Negligent Medical Treatment (mal practice)


 If the doctor makes a situation worse the original negligent defendant(he is the
proximate cause) will still be liable for all intervening negligent mal practice
cases that has happened after the injury that the defendant has caused because it
is foreseeable that a doctor sometimes might make a situation worse instead of
better. Some people will make errors in every profession, it is common but it
won’t happen all the time. It is not surprising so therefore it is foreseeable.
 That doctor will also be liable if he gets sued because he has not conformed to
the custom of other physicians

 2) Intervening Negligent Rescue


 Original defendant will still be liable if a rescuer injures the defendant more than
his original injuries.
 The courts have said that when you negligently hurt someone it is foreseeable
that rescuers will be attracted and sometimes they may make things worse rather
than better.
 It is foreseeable that the rescuers could make things worse, so the original
defendant will be liable and pay for it.

 3) Intervening Protection or Reaction Forces


 Original D will be liable if intervening act was caused by others trying to protect
themselves or a reaction of fear based on the original defendant’s negligent act
 Example 1: D runs a red light, enters an intersection crowded with
pedestrians. D hits a pedestrian and breaks his leg. The other
pedestrians start freaking out & in order to protect themselves,
in reaction they stampede to get out of the way & 1 of these
people steps in the face of the man with the broken leg, who
got hit by the car leaving the man with also a disfigured face.
Driver is liable for both the broken leg & the disfigured face.

 4) Subsequent Disease or Accident


 Example 1: Defendant runs a red light, hits a pedestrian and breaks his
leg. Pedestrian gets a cast and crutches, however because
the pedestrian has never used crutches before he falls and
breaks his arm. The defendant will be liable for both
because it is foreseeable that if you leave someone in a
weekend condition they may have that second accident and
it’s only fair that D should pay.

Practice Question
 Jenny and Pamela were having lunch at Diver Dan’s Seafood Restaurant. Jenny ordered a shrimp
cocktail appetizer. Diver Dan’s had failed to store the shrimp in a reasonable fashion and it had
become infected with bacteria. Shortly after finishing the shrimp Jenny told Pamela she was feeling ill
and excused herself to go to the bathroom. Sometime later Pamela becomes concerned and decided to
check on her. When entering the bathroom, Pamela sees Jenny inside throwing up. Concentrating on
Jenny, Pamela did not see a puddle of vomit that Jenny had left on the tile floor of the bathroom just
seconds earlier. Pamela slipped in the puddle, fell and broke her arm.
 If Pamela sues Diver Dan’s for her broken arm, will she recover? Why or why not?
o Pamela did not have a valid claim because there was no proximate cause
o This is a freakish occurrence

 When looking at a freakish occurrence, look at the risk associated with the breach
 The breach was serving tainted food,
 What are the risks of serving tainted food?
o The risk is that people will get ill, but what happened here is a broken bone
o Broken bones do not normally result when serving people contaminated or rotten food
o This was an unforeseeable outcome and therefore the court felt that liability would be
unfair and exonerated the restaurant

Module 3B: Negligence- Damages


 Harm suffered
 In order to receive damages in Negligence plaintiff must have suffered a detriment
 The plaintiff must show actual harm or injury to complete the prima facie case
 The plaintiff can recover economic damages (medical expenses) and noneconomic
damages (pain and suffering)

Eggshell Skull Doctrine

 Under the Eggshell Skull Doctrine, once the elements have been established the plaintiff
will recover for all harm suffered, even if its surprisingly great in scope
 You take your plaintiff as you find your plaintiff
 If someone has a frailty of some kind, you are still liable
 Applies to every tort on exam, universal principle of tort law even to intentional torts

Module 3B: Negligence- Defence


 Historically we had a doctrine called contributory negligence, assumption of the risk
 These doctrines are minorities in the US and are only used in some jurisdictions
 The ones that are mostly used on the test is the rule of comparative negligence
 Comparative negligence comes in 2 schemes, they are pure comparative negligence and
partial comparative negligence
 Almost all states have rejected the rule that a plaintiff’s contributory negligence will
totally bar her recovery

Comparative Negligence

 The defendant must show that the plaintiff failed to exercise proper care of his own safety
 Proper care means reasonable prudence
 The amount of care that you should exercise to protect yourself from getting hurt is exactly the
same amount of care as you should exercise from preventing others from getting hurt, the care of a
reasonably prudent person
 Proper care can also include a self-protected statute

 Example of a self-protect statute: the law the forbids J-walking, this statute is to protect you from
your own foolish act because that would be a way for you to get hurt if you don’t obey the law
 Plaintiffs damage recovery can be reduced based on their fault and there is no legal rule that will
assign the reduction of those numbers (%), it can be any %.
 The % is based on fault that has been assigned to them by the jury after they weigh the negligence
of the 2 parties, this is a damage reduction
 Example 1: Defendant offers evidence that plaintiff failed to exercise
proper care for their own safety. The jury will then be
instructed to weigh the carelessness of the 2 litigants and
to assign each litigant a percentage number. The plaintiff’s
recovery is reduced based on P’s percentage of fault that
has been assigned to them by the jury, this is a damage
reduction which is the consequence of the P’s fault and
there are no legal rules that set a certain %.
 Example 2: P J-walked and got hit by D a drunk driver. P sustained
$100,000worth of damages. The jury said that P the J-
walker was 20% at fault and D the drunk driver was 80%
at fault. Therefore, P will only recover $80,000 as the
damages were reduced by his own damages of fault.

Pure Comparative Negligence (default on the bar exam, unless they instruct you otherwise)

 In this system we go strictly by the numbers


 Even if the plaintiff is assigned the higher number of faults, the plaintiff still goes home
with some recovery even if she was primarily 90% at fault.

Partial/modified Comparative Negligence

 A plaintiff’s fault 50% or less reduces recovery


 A plaintiff’s fault over 50% is an absolute bar and the plaintiff gets $0, they are barred
from recovering anything
Module 4: Strict Liability
 There are 3 causes of action in strict liability
 Will be listed by importance
 Strict liability for animals
 Strict liability for abnormally dangerous activities
 Strict liability for Products

Strict Liability for Animals


 Animals cases requires you to distinguish between 2 broad groups of animals:

 1) Domesticated Animals
 Domesticated animals are house pets and livestock
 House pets are cats and dogs
 Livestock are agriculture animals such as cows, horses, pigs,
ducks, goats, sheep and chickens to produce labor and
commodities such as meat, eggs, milk, fur, leather
 You are not strictly liable for domesticated animals, not strict liability
 If someone gets hurt by any of these animals there is no strict liability and they will have
to establish some negligence on your part

 Exception: If your animal has viscous propensities and these are known to you, NOW you
will be strictly liable, it is strict liability for keeping a domesticated animal with
known viscous propensities. This viscous propensity has to be to this main
species not to the entire specie. However, animals that cause a viscous
propensity does not apply to trespassers on your land, it only applies to public
people on the street and to social guest at your home or business customers at
your place of business.
 All mules of donkeys will kick occasionally, that his behaviour common to
a mule and no viscous propensities
 All bulls will put their heads down and charge occasionally, that does not
mean it has viscous propensities
 A dog that bites humans IS a viscous propensity because dogs have been
breaded to not do that, but some do. You could have known if once your
dog bites someone, once it does you know that your dog is the rare dog
that bites people and if you continue to keep that dog you will be strictly
liable if he bites again.
o The first bite puts you on notice, second bite strict liability

 2) Wild Animals
 If dealing with wild animals, then there is strict liability
 No knowledge or first bite needed
 Safety precautions are irrelevant because its strict liability
 Wild animals- elephants, monkeys, lions, tigers, bears
Strict Liability for Abnormally Dangerous Activities

 Abnormally dangerous activities is an activity that meets a 2-part test:

 1) Activity creates a foreseeable risk of serious harm, even when reasonable


care is exercised

 Reasonable care cannot shrink the amount of risk down to a comfortable


level, the amount of risk will still strict liability no matter what.
 The activity is abnormally dangerous, even when engaged in carefully it
still produces a risk of a bad outcome

 2) Activity is uncommon in the community where it is being conducted

 If everyone is doing it, it might be dangerous but by definition it’s not


abnormally dangerous because everyone is doing it so its normal

There are 3 Types of Abnormally Dangerous Activities:


 Anything involving:

 1) Explosives

 Dynamite, PNT(explosive material), nitrogen, plastic(soft & hand-mouldable sold


form of explosive material), nitroglycerin or any explosive materials

 2) Highly Toxic Chemical or Biological Material

 Biological material is usually blood, urine, human tissue, organs, cells,


tissues, living and dead people. Some can pose a threat such as virus or
toxins from a biological source that is affected and can affect/threat to
human health.
 Example 1: In a lab they are trying to find a cure for
the Ebola vaccine. In the lab they have
live Ebola cultures, that is an abnormally
dangerous activity

 3) Radiation or Nuclear Energy

 Safety precautions are irrelevant(it doesn’t matter if they are safe), it is always strict
liability
 Nuclear power plants emit radiation
 Demolition companies use radiation
Module 4: Strict Liability for Products
 In a Strict Liability case the plaintiff must satisfy the 4 elements
 Strict liability refers to both industrial and consumer products
 If someone gets hurt by a product they will have multiple possible causes of actions and
possibly multiple defendants such as a wholesaler, manufacturer and retailer
 There is not requirement of privity of contract to bring a strict liability claim, which
means you can sue people that you did not deal with directly

 1) Consumer Products

 It can be a small appliance you have in your home such as:


 A toaster, blender, waffle iron or it can be a car, boat, motorcycle
 It can be a food product such as a can of tuna fish, bottle of olives

 2) Industrial Products

 It can also be something that you encounter in a factory:


 A forklift truck, drill press or industrial machinery that lack safety
features such as guards to prevent your hand from getting mangled

 Example 1: Paul has a blender at home and it hurts him


 Paul may be able to bring a negligence claim against the company that manufactured
the blender.
 Maybe the manufacturer put it together and did something careless such as
forgetting to tightening some screws.
 If Paul does not have direct evidence of what they did wrong he may be able to rely
on Res ipsa loquitor.
 Paul may have an action against them for the breach of warranty of merchantability
that rises under Article 2 of the UCC
 If they lied about the product, Paul may have a claim for misrepresentation or fraud
 Paul may have a claim for strict liability

 There is many different situations that could be tested, make sure YOU analyze the
under the right certain theory that the examiners asks about

If the call of the questions says:


 Example 2: Patricia sued the department store for negligence
 If you use the strict liability analysis you will get it wrong
 If it is strict liability it has to satisfy the elements
There are 4 Elements that a Plaintiff Must Show in Strict Liability

 In strict liability the plaintiff must show that:


1. The Defendant is a Merchant
2. The Product Must be Defective
3. The Product has Not Been Altered Since It Left the Defendants Hands
4. The

 1) The Defendant is a Merchant

 You can’t rely on strict liability unless the person you are suing is a merchant
 A merchant is someone who routinely deals in goods of this time

4 Common Mistakes:

 1) A Casual Seller is not a merchant and cannot be strictly liable. A casual seller
is you or me selling used goods online or in the newspaper
 If you sell your car online to someone and the breaks fail you may be
liable in another tort but not in strict liability

 2) Service Providers sometimes make products available incidental to the


service, but they are not considered merchants of those products, therefore they
are not strictly liable if something goes wrong with the product
 You go to the doctor’s office and sit on a chair and it breaks, the doctor
is not strictly liable because the doctor is not a merchant of chairs, he
is a vendor of professional medical services. You are an invitee and
may have a premises liability negligence claim but not strict liability

 3) Commercial Lessors are people who are in the business leasing stuff. These
people ARE MERCHANTS and can be strictly liable.
 A rental car company. You rent a car to go to Florida and get 5 miles
away when the breaks fail on that rental car.
 The rental car is strictly liable

 4) Parties up the Distribution Chain with whom you did not deal with directly
are also merchants and are strictly liable
 Wholesaler
 Manufacturer
 Retailer

 2) The Product Must be Defective

 The plaintiff must show that the product suffers from a defect
 On an essay, you must analysis the 3 different kinds of defective any one will
allow the plaintiff to move on with the case
3 Types of Defects, in which only 1 needs to be satisfied:

 1. A Manufacturing Defect: (1 in a million)

 A product has a manufacturing defect when it differs from all the others that came off
the same assembly line, in a way that makes it more dangerous then consumers would
expect
 A product has a manufacturing defect when it departs from its intended design that
makes it defective
 If all the other products has 4 bolts and this one has 3 bolts
 If all the other products have a smooth edge and this one has a jagged
edge, it can be a defective product

 In a manufacturing defect, safety precautions do not matter because its strict


liability. Any safety methods they used does not matter, its strict liability.

 2. A Design Defect:

 The law has been extremely controversial, political and unsettled for a long period of time
 Products have a design defect, if costs of the design outweigh the benefits of the design,
such that a reasonable merchant would not have put it on the market for sale
 Cost of the design are usually its tenancy to hurt you
 Benefits of a design can be things like, it makes the product easier to use, or more
comfortable to hold, quicker to use
 Benefits can be things that make the cost of the item lower, they reduce the
manufacturing expenses
 There is always a balance between incorporating safety features that may make the
product heavy or cumbersome or slow or expensive or leaving those safety features off in
order to make the product light and easy to use, but more risky
 So, the test is due to the cost of danger that outweighs the benefit types listed above such
that a reasonable person wouldn’t market the product
 The law of design defect is a strict liability claim, however it is a form of negligence
liability, so you do use reasonable person

 How does the plaintiff establish that the cost outweighs the benefits(more safety in the design)?

o It is by offering evidence of a reasonable alternative design, that passes the 3-part test
o They could have built it better by not making anything negative happen and their failure
to do this is evidence of what they put on the market is unreasonable
3 Part Alternative Design Test:

 1) Its Safer Than the Product Marketed


 They hypothetical alternative design is a safer design

 2) It is Economically Feasible
 The alternative design is about the same price to manufacture
 It has to be cost effective

 3) Its Practical
 It doesn’t make the product difficult to use

 If an alternative design exists then that suggests that the cost of the actual design
outweighs its benefit because there is a better way to build it and that makes the
product defective

 Example 1: In the 80’s the design of baby cribs had vertical slats in
where they were more widely spaced apart then today.
Some babies would be able to stick their heads in the slats,
get trapped, choke and a tragedy would result. This has
happened to Patricia and so she brought a cause of action
that claimed that the crib was defectively designed. It had
a design defect and the costs (the risks) of that product
outweighed its benefits. Pat would show that by saying “I
can show you an alternative way of designing that crib”.
The alternative was it moving the slats closer together, so
that the baby can’t stick its head in-between the slats.

 Does that alternative design meet the 3-part test?


o 1. Is it safer? Yes, no straggled babies
o 2. Is it cost effective/economically feasible? Yes, spend a few more dollars for 1
piece of wood, then just reposition
o 3. Is it practical? Yes, the crib still functions perfectly as a crib. Baby still gets air,
light, visual stimulation, you can still see the baby, touch the
baby and put your hand through the slats to touch the baby
 Therefore, it is a good alternative design and those old cribs were defective

 3. An Information Defect:

 If a product has residual risks that cannot be designed out & if that risk is not obvious or
apparent to the users/consumers, the product is defective if it lacks adequate warnings &
instructions
 If the risk is hidden the product needs a warning and the warning must be adequate
 Not all warnings are created equal
o Every product has some residual/remaining risk that can’t be designed away
such as a knife. A knife can cut and hurt someone, if you make it dull it will
not function as a knife and that’s not practical.
o Cutting yourself is an obvious risk, so you don’t need a label that says
“warning do not run tongue around blade” that is unnecessary

Issues with Adequate Warnings:

 When looking at warnings, ask if the warning is adequate or it could be easily improved upon

 1. Placement

 Putting a warning on page 8 of the instructions might not be adequate


 Adequate should be on a big red sticker on the point of use

 2. Comprehensibility

 If the wording is to complicated or in English only as it should be bilingual


 Perhaps the warning should not only use wording but also icons for people who
don’t know how to read

 3. Risk Mitigation Information

 Sometimes warnings are not enough unless it is accompanied by risk mitigation


 Warning should show how to mitigate the risk if there a logical way
o Like when using a pesticide, the warning label says “warning toxic fumes”
and then it should say “wear mask while using”(mitigating the risk)

 3) Plaintiff Must Show Product Not Altered Since It Left Defendants/Manufacturers Hands

 There is a presumption that the product has not been altered, so long as it moved in ordinary
channels of distribution
 As from an ordinary retailer, who got it from the wholesaler, who got it from the manufacturer
 There is a presumption that the product that you buy from the retailer is in the same shape as
when it left the factory floor, as it moved in ordinary channels of distribution
 The plaintiff can rely on the presumption to satisfy the element and then it’s up to the defendant
to offer contrary evidence, if any

 This presumption does not apply to items purchased used, as it could have been
abused/misused by the previous owner in many different ways
 If a used product hurts you, it is on you to show that it is in the same condition
that is was when it left the manufacturer hands, if you go to sue the manufacturer

 4) Plaintiff Must Show at the Time of Injury he was making Foreseeable Use of Product

 Plaintiff has to be making a foreseeable use of the product at the time of injury
 A foreseeable use is not necessarily an appropriate or proper use
 Many missuses, many inappropriate uses are nonetheless entirely foreseeable
 Do not think the test is if the product is being misused, the test is whether the use is foreseeable
 The test it foreseeable use not proper use
 In foreseeable use, there are people that are wacky that can use products in unforeseeable ways
like if you get a lawn mower and try to cut your hair with it that is an unforeseeable use and will
be no liability for it. (jackass movie)

 Example 1: Pete needs something in a high shelf in his home, to get it he stands up on a
chair and then the chair collapses because it can’t support his weight. He brings
a claim against the chair manufacturer. It is a manufacturing defect, it is the 1 in
a million bad chair that can’t support his weight. The manufacturing company
says that he was not using the chair in the correct manner as chairs should not be
used as step ladders, they should be used as seating only. The courts said that’s
irrelevant. The question is whether it is foreseeable that people will stand up on
chairs. The answer on foreseeable is, yes as everyone does it.

Strict Liability Defense of Comparative Responsibility:


 All 3 of the Strict Liability Torts only have 1 concept of an available defence
 The defence is similary to Comparative Negligence but it is a different name
 The defence is called Comparative Responsibility

 If the defendant can prove the plaintiff in strict liability case was careless, foolish, arrogant,
cocky& did not take proper effort to protect his/her own safety & confronted known risk then:
 Jury will assign some percentage of responsibility to P & P’s recovery will be reduced.

 Example 1: Your neighbour has a tiger and you go with a stick and poke it through
the fence. The tiger gets annoyed and attacks you. You try to sue the
neighbour for strict liability for keeping a tiger, but you will be charged with
a very high percentage of the fault because you were taunting the animal
 Example 2: Your toaster was sparking earlier but you were very keen on having a
piece of toast so you use it anyways. The toaster then explodes and you try to
sue the manufacturer for strict liability for a manufacturing defect, it was that
1 in a million bad toaster. You will be charged with a very high percentage of
fault because you knew it was sparking so why would you use it.

Practice Question:

 Delta makes toasters & sells to retailers for distribution to consumers. Pamela bought a Delta brand
toaster from SuperDeal electronics store & when she got home, plugged it gave her a severe shock. She
sued Delta, the manufacturer, in strict liability. At trial, she offers evidence of her injury. Delta offers
evidence that the specific toaster Pamala purchased had been knocked off of a high shelf by an
employee at the electronic store, who placed it back on the shelf. Delta also offers evidence that an
examination of the toaster showed that some safety circuits were disconnected &an expert testified that
this was likely due to the result of the product having been dropped from a significant height.

 If all this evidence is admitted at trial and the case is sent to the jury, will Pamela be able to win
her strict liability claim against Delta? Why or Why not?

o Pamela will not succeed as this product has been altered since it left Deltas control
o Normally she could rely on presumption that not been altered, but defendant rebutted the presumption
with extensive evidence that it had been dropped by retailer & dropping caused damage & directly
traceable to nature of injury as involved safety circuits
o Evidence rebuts presumption & defect was introduced after it left their hands, they are not responsible
 If held Delta liable, with this question Delta would have turned into an insurance company.
 Delta strictly liable, but only for things that happened on their watch. Toaster was altered, so off the hook.
Module 5: Nuisance (1 question not heavily tested)
 A Nuisance is a tort that involves real estate/property
 Nuisance is defined as unreasonable interference with plaintiff’s ability to use & enjoy property
 Unreasonable does not mean that the defendant is necessarily acting unreasonably, what it
means is that the amount of interference is unreasonable
 It means that the defendant is doing something that makes a nearby land owner
miserable, D has rendered P’s condition intolerable

 Many not all nuisances cases involve inconsistent land use, unfortunate position of 2 commercial
enterprises, sometimes business & residence that should have never have been near each other

 Example is a smoking factory next to a sanitarium for people suffering for asthma, these 2
cannot be sole locating because of the smoke coming from the factory would make it
difficult to run that sanitarium with the people with asthma, as it would make them worse.

 Example 1:Heavy metal recording studio next to recovery facility for nervous break downs
 Example 2: A plant nursey sold bulk cow manure as fertilizer next to outdoor cafe
 Theses 2 business should not be next to each other & cause a nuisance to one another

 A Nuisance can also involve spite or inconsideration


 You deliberately shine lights onto your neighbour’s property, your neighbour may have a cause
of action for nuisance. You are being malicious & neighbour can never sleep because lights are
bright even with drapes/curtains neighbour’s house is still lite up
 Your neighbour is inconsiderate & throws loud parties all night/day can be a nuisance
 Your neighbour is running an illicit operation, drug supermarket. Strange people coming &
going, cars parked on the street, weird noises & occasional gun shots. Could be a nuisance

The key question is a balancing of the interests

 Defendant is entitled to use his land as he wishes, if it’s a lawful purpose. If you want to have a
recording studio for a heavy metal band & not in any ordinance, you are entitled to do that BUT
 Your neighbour should also be free from misery, your neighbour should not be subjected to such
a position that his property becomes worthless and has to vacate it
 If you’re going to make the parcel next door uninhabitably, then buy parcel next door because
you can’t do this as it is considered a nuisance
 We are all obligated to put up with minor inconveniences, don’t rise to the level of a nuisance
 Your neighbour mown the lawn 8am Sunday & you like to sleep in Sunday it is
not enough to rise to a level of a nuisance
 Live across from a church & hear church bells. You work from home &
disturbs your concentration, not enough to rise to a level of nuisance
 A level of nuisance has to be something that makes your life intolerable
 What do you do if you get a Nuisance Question on the Multistate?

o You look for the choice that imbodies the legal rule,that says:
o “plaintiff will win if defendants conduct interfered with ability to use &
enjoy property to an unreasonable degree” or
o “ Plaintiff will win if the balance of interest tips to the plaintiff’s favour “

Module 6: Miscellaneous Topics Including Vicarious Liability


 Topics and categories go in order as most important and will likely be on the test

Vicarious Liability

 It’s possible to hold the direct tortfeasor liable but also a secondary party liable as well
 The secondary party will usually be liable because of a relationship
 Vicarious liability is predicated on 4 different relationships

4 Categories of Relationships in Vicarious Liability:

 1. Employer  Employee

 Vicarious liability if employee commits a tort, employer will be liable if tort was
committed “within the scope of employment”
 What is the scope of employment?

 Is an Intentional Tort Within the Scope of Employment?


 Example 1: Diane runs a flower business & employee delivery driver named.
Dave runs a red light while delivering flowers, Diane as the employer
is vicariously liable for her employee Dave running the red light.

 Example 2: Supermarket has Don as employee, he is stalking cans on the


shelf. A customer says to Don “excuse me where to you keep the
canned pasta?” Don then turns around and kicks the customer in the
growing’s.

 Is the Supermarket Liable for Dons Intentional Tort?


o No, intentional torts are outside the scope of employment. BUT it’s not an absolute rule,
there is circumstances where these intentional torts come back into the scope of
employment such as these situations:

Intentional Torts that are in the Scope of Employment


 1. If Job Involves Use of Force, Then Misusing the Force is Within Scope of
Employment

 Jobs that involve use of force are store security guards, night club bouncers
 It gives the nightclub the incentive to hire calm people because if you are
going to allow them to use force we rather not you hire them, so we will
make you liable for any tort they have committed

o A night club bouncer loses his cool as see a romantic rival he starts
wailing on him & starts punching him, committing a battery. The
nightclub will be vicarious liable for the bouncer’s battery.

 2. If the Job Leads to Friction or Tension

 It also gives creates in incentive to hire calm minded people for that kinds of
work because if you are going to allow them to use force we rather you not hire
them, so we will make you liable for any tort they commit
o The guy who repossesses your automobile if you don’t make payments, if
that guy loses his cool and punches a customer. The bank or the finance
company that employed him, they are going to be vicariously liable

 3. Any Intentional Tort That is Done to Serve the Bosses Purpose, will be Within
the Scope of Employment

 The store security guard, who starts randomly detaining and searching
customers to cut down on shop lifting, is committing false imprisonment.
 That store will be vicariously liable because that was done to serve the
stores purpose

 2. Hiring Party  Independent Contractor

 General rule is no liability in torts for the independent contractor, but there is an
exception
 If you hire an independent contractor to work on your business premises, you will be
vicariously liable if that independent contractor hurts a customer or an invitee while at
work, working for you.
 You owe a non-delegable duty of care to your customers, you remain responsible to them
 Example 1: You hire painters to paint your convince store, they
have to work while the store is open because the store
is open 24/7. On route to the store they run a red light
and hit a pedestrian, you are not liable. The painters are
independent contractors they are not your employees,
so no vicarious liability. However, once they get to the
store they set up a ladder and put a tub of paint on the
shelf and then accidently knocks it over and hits a
customer in the head. You will be vicariously liable for
that.

 3. Automobile Owner  Automobile Driver

 An owner of a car is not vicariously liable for the torts of a driver


 If I lend you my car, I am not responsible if you go out and get drunk and drive
drunk, or if you speed or if you run a red light
 The owner of the car will be vicariously liable IF I ask you to take my car to do an
errand for me because we are in a different relationship, which is Principle  Agent
o You are always vicariously liable for the torts of your agents

 4. Parent  Child

 No vicarious liability at all, parents are not liable for the torts of their kids
 If you get hurt by a kid, you sue the kid. Kids are covered by insurance and
sometimes have their own assets, but the parents are no vicariously liable.

Multiple Defendant Issues:


 Comparative Contribution Rule- where the defendant gets a % of their money back
 Indemnification- where the defendant gets the full reimbursement
 Non-manufacturer- In a strict liability claim a wholesaler and retailer can be fully
indemnified by the manufacturer

 Example 1: Assume that the plaintiff has sued multiple defendants


Ali, Bill and Carl. The plaintiff will have won the claim
and recovered all the money from 1 of the defendants. It is
a $100,000 judgement and the plaintiff got it all from Ali.

 Does Ali have any rights against his co-defendants Bill and Carl to get some of that
money back? Is there a right of reimbursement?

o Yes, Contribution is available and the amount is set by percentages signed by the
jury
o The jury gives each defendant a percentage and we just follow those numbers

 If the jury said:


 Ali was 10% at fault and
 Bill was 30% at fault and
 Carl was 60% at fault
 Then Ali gets $30,000 back from Bill and
 $60,000 back from Carl

 This will happen under the Comparative Contribution Rule

There are 2 Situations Where Defendants Can Get all their Money Back
 Full reimbursement and not only a percentage

 1) Indemnification:

 Arises in situations where defendant is vicariously liable & defendant paid plaintiff.
 vicariously liable party can get full reimbursement or indemnity from active tortfeasor
 2) A Non-Manufacturer

 Wholesaler/retailer can get full indemnification from manufacturer in strict liability claim

 Example 1: If P sues Home Depot and Black & Decker because P bought a
defective power tool and P wins against both of them. P
recovers all the money from Home Depot. Home Depot can then
get an indemnity from Black & Decker. Home Depot goes to
Black & Deck and says “ I just paid the plaintiff now give me
that money back”. That has the effect of shifting the loss up the
chain so that it rests on the shoulders of the manufacturer.

Loss of Consortium :

 Whenever married person injured valid tort claim, spouse also gets a separate independent
claim
 The uninjured spouse gets a second separate independent claim
 The claim is derivative(it is based on another source), that means any defences that can be
asserted against the plaintiffs injured spouse can also be asserted against the uninjured
consortium spouse

 If an argument of consent & also raise that against the consortium party
 If an argument of comparative negligence & also raise that against consortium party

 Why is There a Loss of Consortium Claim? Why Give the Spouse a Remedy?

o It is to allow recovery of 3 types of damages that would otherwise be uncompensated

3 Types of Damages That Would Otherwise Be Uncompensated are:

 The loss of consortium claim (spouse) can recover for:

 1. Loss of Household Services

 Argument is with spouse injured no one to do dishes, laundry, cleaning,


cooking, taking the kids to school & they might need to hire someone to do
these things & get money for it.

 2. Loss of Society

 Loss of society is loss of companionship


 The argument is that with my spouse injured the other spouse has no one
to talk to and you do get money for that.

 3. Loss of Sex

 Argument is with spouse injured, can’t have sex with them. Also get money for
this.

You might also like